9/7-MBE CONTRACTS & SALES

अब Quizwiz के साथ अपने होमवर्क और परीक्षाओं को एस करें!

Question 6011 On May 10, the coach of a youth league baseball team sent a letter to a supplier asking the supplier to promptly ship 20 red jerseys to him. On May 15, the supplier received this letter and sent the coach a reply letter accepting the offer. On May 16, the supplier realized that he had no red jerseys with which to fill the order, and sends the coach 20 blue jerseys with a note that the blue jerseys were tendered as an accommodation. The coach received the jerseys and accommodation note on May 18, and received the supplier's acceptance letter on May 19. On May 20, which of the following is a correct statement of the parties' legal rights and duties? Answers: Correct Answer: The coach can either accept or reject the blue jerseys and, in either event, recover damages, if any, for breach of contract. The coach can either accept or reject the blue jerseys, but if he rejects them, he will thereby waive any remedy for breach of contract. The supplier's shipment of nonconforming goods constituted an acceptance of the coach's offer, thereby creating a contract for the sale of the blue jerseys. The supplier's shipment of the blue jerseys constituted a counteroffer.

Answer choice A is correct. A seller's shipment of nonconforming goods with a notice of accommodation does not constitute an acceptance and breach, but rather a counteroffer, which the buyer is free to either accept or reject. However, an offer calling for prompt shipment can be accepted either by a prompt promise to ship or by the prompt shipment of goods. Under the mailbox rule, an acceptance is effective when mailed. Consequently, on May 15 when the supplier mailed his acceptance to the coach, a contract for 20 red jerseys was formed, even though the coach did not receive this acceptance until May 19. With regard to the May 16 shipment of blue jerseys by the supplier that the coach received on May 18, the coach may accept or reject these jerseys as nonconforming goods and, in either event, recover damages, if any. Answer choice B is incorrect. Although a seller's shipment of nonconforming goods with a notice of accommodation does not constitute an acceptance and breach, but rather a counteroffer, which the buyer is free to either accept or reject, this is only true if the seller has not already accepted the buyer's offer. Here, the supplier had earlier mailed its acceptance of the coach's offer to supply red jerseys, so the supplier's shipment of the blue jersey constitutes a breach of contract. Consequently, the coach may reject the blue jerseys without waiving any remedy for breach of contract. Answer choice C is incorrect because the supplier's shipment of the blue jerseys did not constitute an acceptance of the coach's offer, but a breach of the contract that had already been formed for the shipment of red jerseys. Answer choice D is incorrect because, although typically an accommodation shipment constitutes a counteroffer rather than an acceptance and breach of the contract, here the shipment, despite being designated as an accommodation, cannot be a counteroffer because the supplier had already accepted the coach's offer.

Question 6337 A collector of rare cars sent a letter to a well-known car enthusiast on January 2 regarding the possible sale by the collector of a limited-edition car. On January 3, the car enthusiast sent a letter to the collector, offering to purchase the limited-edition car for a price of $35,000. The letter also stated that the car enthusiast reserved the right to cancel the contract before January 10. Upon receiving the car enthusiast's offer on January 5, the collector immediately mailed an acceptance of the offer to the car enthusiast. On January 6, the car enthusiast sent a fax to the collector cancelling the contract. The car enthusiast received the collector's acceptance on January 7. The collector sued the car enthusiast for breach of contract. Which party is likely to prevail? Answers: The car enthusiast, because his letter on January 3 was not a valid offer. The car enthusiast, because the collector's acceptance arrived after he canceled the contract. The collector, because the car enthusiast's offer was irrevocable. The collector, because his acceptance was effective on January 5.

Answer choice A is correct. A statement is an offer only if the person to whom it is communicated could reasonably interpret it as an offer. It must express the present intent of a person to be legally bound to a contract. The primary test of whether a communication is an offer is whether an individual receiving the communication would believe that he could enter into an enforceable deal by satisfying the condition. In his letter to the collector offering to purchase the car, the car enthusiast reserved the right to cancel the contract before January 10. Accordingly, it would not be reasonable for the collector to believe that he could form an enforceable deal with the car enthusiast by accepting the offer prior to January 10. Therefore, the collector's attempted acceptance was not effective because there was not a valid offer. Answer choice B is incorrect. Under the mailbox rule, even if the offer had been valid, the collector's acceptance would have been effective upon mailing, not receipt. Here, the fact that the car enthusiast cancelled the contract prior to receiving the collector's acceptance has no effect on the outcome because there was no valid offer to accept. Answer choice C is incorrect because there was no consideration to make the offer irrevocable. Moreover, the car enthusiast specifically reserved the right to revoke the offer (cancel the contract). Answer choice D is incorrect. Although the collector's acceptance would have been effective when it was mailed on January 5 under the mailbox rule, there was not a valid offer for the collector to accept at that time.

Question 6023 In January, a local farmer contracted with a chef to sell the chef a specified amount of local organic tomatoes to be delivered on August 1. On June 15, the farmer called the chef to tell him that part of his crop was infested with tomato fruitworms, and he was unsure that he would be able to deliver the full amount requested by August 1. The chef told the farmer that it was absolutely essential that he receive those tomatoes on time to make organic tomato sauce for a restaurant scheduled to open in August. The farmer assured him that he would do his very best to save the crop and deliver by August 1. Does the chef have valid legal grounds to cancel the contract and order tomatoes from another source? Answers: No, because the farmer did not state unequivocally that he could not deliver the tomatoes on time. No, because the farmer still had more than 30 days in which to deliver the tomatoes. Yes, because the farmer committed an anticipatory repudiation of the contract by causing the chef to feel insecure about the performance. Yes, because the farmer failed to provide adequate assurances to the chef.

Answer choice A is correct. Anticipatory repudiation occurs when there has been an unequivocal refusal of the buyer or seller to perform, or when reasonable grounds for insecurity arise with respect to the performance of either party, and the other party fails to provide adequate assurances within a reasonable time (not to exceed 30 days under the UCC). Mere expressions of doubt as to a party's ability to perform do not constitute an anticipatory repudiation. Therefore, the farmer has not repudiated, and the chef is still bound to the contract. Answer choice B is incorrect because the 30-day time period is only relevant if the chef had asked the farmer for assurances, which he did not. Answer choice C is incorrect because, while expressions of doubt by a party as to its ability to perform may give the other party the right to demand adequate assurances, they do not amount to the affirmative manifestation of intent required to constitute an anticipatory repudiation. Answer choice D is incorrect because, although anticipatory repudiation occurs under the UCC when a party creating reasonable grounds for insecurity fails to provide adequate assurances within 30 days of demand for assurances, this requirement for assurances only applies where demand has been made. Here, the chef reiterated that he needed the tomatoes by the agreed-upon date, but he never demanded assurances from the farmer.

Question 6400 A manufacturer of t-shirts contracted with a new clothing store to sell the store 1,000 t-shirts per month for a period of two years. The clothing store's signature color for their clothing was an orange-tinted red color, called coquelicot, which is very difficult to replicate on a consistent basis. The contract specified that any t-shirts that were not coquelicot could be returned, but it was silent with regard to the return of any t-shirts for other reasons. One year into the contract, the store decided to switch to coquelicot-colored baseball hats instead of t-shirts. The store returned the most recent shipment of t-shirts to the manufacturer and demanded a refund. The manufacturer refused to grant the refund, and the store sued the manufacturer for damages. At trial, the manufacturer introduced the contract, which clearly stated that t-shirts that were not coquelicot could be returned. The store then attempted to introduce evidence that it had returned t-shirts for other reasons to the manufacturer in the past and received a refund. Is this evidence admissible? Answers: Yes, because the evidence is relevant to show that the manufacturer had accepted the return of coquelicot-colored t-shirts in the past. Yes, because the evidence can reasonably establish the parties' course of dealing on this issue. No, because the evidence regarding the return of the t-shirts violates the parol evidence rule. No, because the express term in the contract regarding the return of t-shirts takes precedence over the course-of-performance.

Answer choice A is correct. Course-of-performance evidence is admissible under the UCC to explain or supplement a contract. A course of performance is a sequence of conduct that is relevant to understanding an agreement between the parties if: (i) the agreement involves repeated occasions for performance by a party, and (ii) the other party accepts performance without objection and with knowledge of the course of performance. Here, there were monthly purchases of t-shirts and evidence that the manufacturer had accepted returns of coquelicot-colored t-shirts in the past. Therefore, this course-of-performance evidence is admissible. Answer choice B is incorrect because course of dealing refers to conduct between the parties based on another contractual relationship. Here, because there is only the one contract between the two parties and the evidence relates to the performance of that contract, rather than another contract, the evidence does not relate to their course of dealing, but rather their course of performance. Answer choice C is incorrect because the parol evidence rule does not prevent the introduction of evidence regarding conduct by the parties after the written contract was executed. Answer choice D is incorrect because the agreement was silent as to the return of coquelicot-colored t-shirts. Thus, course-of-performance evidence is admissible to explain how the parties viewed the right of the store to return such t-shirts.

Question 7035 On June 1, a cereal manufacturer hired an engineer to repair its only cereal-packaging machine by June 15. On June 5, a granola company contacted the cereal manufacturer to offer them a contract to produce 10,000 boxes of the granola company's most popular recipe to fill an order by a chain of grocery stores. The granola company needed to produce 10,000 boxes by June 20 to meet its contractual duties to the chain of grocery stores and to avoid high damages from lost sales. Believing it would have time to fill the order if the packaging machine was repaired by June 15, the cereal manufacturer agreed to a liquidated damages clause for $1,000 for each day's delay in delivery of the boxes after June 20. The engineer was inexcusably three days late repairing the packaging machine, and as a result, the cereal manufacturer was three days late in delivering the 10,000 boxes to the granola company. The cereal manufacturer paid the granola company $3,000 under the terms of their contract. The cereal manufacturer and the engineer knew on June 1 that under ordinary circumstances, the cereal manufacturer would suffer virtually no damages from a three-day delay in the repair of the packaging machine. If the cereal manufacturer sues the engineer for breach of contract and demands $3,000 in consequential damages, is the cereal manufacturer's request for damages likely to succeed? Answers: No, because the damages were unforeseeable on June 1. No, because time was not of the essence in the contract between the cereal manufacturer and the engineer. Yes, because the engineer's inexcusably late performance was the but-for cause of the cereal manufacturer's damages. Yes, because the liquidated damages clause was reasonably related to the sales lost by the granola company.

Answer choice A is correct. Damages are recoverable if they were the natural and probable consequences of breach, or if they were in the contemplation of the parties at the time the contract was made, or if they were otherwise foreseeable. Here, neither the cereal manufacturer nor the engineer had any reason to expect on June 1 that late performance on their contract could result in such high damages. Therefore, the cereal manufacturer cannot recover these consequential damages from the engineer. Answer choice B is incorrect because, regardless of whether time was of the essence on the engineer's contract with the cereal manufacturer, the cereal manufacturer cannot recover unforeseeable consequential damages from the engineer. Answer choice C is incorrect because the fact that the engineer's delay was the but-for cause of the cereal manufacturer's damages is insufficient to allow the cereal manufacturer to recover these unforeseeable consequential damages from the engineer; it can only recover consequential damages that were reasonably foreseeable at the time the contract was made. Answer choice D is incorrect. Even if the liquidated damages clause in the contract between the cereal and granola companies was reasonable and enforceable, that is not sufficient to make the damages suffered by the cereal manufacturer due to the engineer's delay reasonably foreseeable on June 1. Therefore, this fact is not relevant to the cereal manufacturer's ability to recover from the engineer.

Question 4355 A recent college graduate offered to buy all of the computers from a struggling online retailer for which he had been an intern during college. The terms of the written agreement were such that the graduate would pay $10,000 for a "reasonable number of computers, as the closing retailer no longer needed them." Due to the graduate's internship with the retailer, he knew that there were 50 computers in the office and knew that nearly all of them were unused, so he believed he would receive all 50 computers once the retailer closed. He gave the retailer a check for $10,000 and in return, took ten computers from the office that day. With the help of the $10,000 and a sudden upswing in sales in the online retail market, the retailer became profitable. When the graduate demanded the remaining 40 computers, the retailer refused. Instead, the retailer returned the $10,000 to the graduate and demanded the return of the ten computers in the graduate's possession. The graduate sued the retailer for breach of contract. The retailer moved to dismiss and argued that no valid contract existed. How should the court rule? Answers: Grant the motion, because there was no agreement as to quantity. Grant the motion, because the retailer's increased profitability constituted a supervening event. Deny the motion, because the court may supply missing terms in a contract. Deny the motion, because the parties both had the objective intent to form a contract.

Answer choice A is correct. For a contract to exist, the terms of the contract must be certain and definite, or the contract fails for indefiniteness. Under common law, all essential terms - the parties, subject matter, price, and quantity - must be specified in the agreement or the contract fails. Under the UCC, a court may supply missing terms, but a contract must still specify the parties, subject matter, and quantity. Here, the contract is governed by the UCC, and because it is missing the key term of quantity, it is therefore invalid. Answer choice B is incorrect because the increased profitability would not discharge the retailer's duty to perform if performance had been required. Answer choice C is incorrect because under the UCC, the court may not supply a missing quantity term. Answer choice D is incorrect because although the parties intended to form a contract, they failed to adequately define the terms of the contract.

Question 7034 A bicycle vendor contracted with a supplier to purchase 60 identical helmets to be delivered in six monthly shipments of 10 helmets each. The first four shipments arrived in perfect condition. Upon receiving each shipment, the vendor accepted the helmets and made a payment to the supplier for one-sixth of the total price under the contract. The fifth shipment arrived with only four helmets and a note from the supplier that read as follows: "Six of the helmets in this shipment were damaged when we received them from the manufacturer. We will deliver six replacement helmets within two business days." The four helmets received were in perfect condition, and the vendor has an adequate supply of helmets remaining to prevent any loss of sales in the next two business days. Which of the following is true regarding the vendor's rights under the contract? Answers: The vendor must accept the four helmets and is not entitled to cancel the rest of the contract. The vendor must accept the four helmets, but is entitled to cancel the rest of the contract based on the breach. The vendor may accept or reject the helmets, but is not entitled to cancel the rest of the contract based on the breach. The vendor may accept or reject the helmets, and is entitled to cancel the rest of the contract.

Answer choice A is correct. If the seller makes a nonconforming tender or tenders nonconforming goods under one segment of an installment contract, the buyer can reject only if the nonconformity substantially impairs the value of that shipment to the buyer and cannot be cured. If the seller makes adequate assurances that he can cure the nonconformity, then the buyer must accept the shipment. Here, the nonconforming shipment does not substantially impair the value of that shipment to the vendor, and the supplier has given adequate assurances that the nonconformity will be promptly cured. Therefore, the vendor must accept the shipment, and answer choices C and D are incorrect. When there is a nonconforming tender or a tender of nonconforming goods under one segment of an installment contract, the buyer may cancel the contract only if the nonconformity substantially impairs the value of the entire contract to the buyer. Because the vendor will likely suffer no damage from this delayed shipment of six helmets, the nonconformity does not substantially impair the value of the entire contract, and the vendor cannot cancel the contract based on this nonconformity. Therefore, answer choices B and D are incorrect.

Question 6357 A college student wanted to purchase a car so he could visit friends at other nearby colleges. After looking at a few different used cars, the college student found one he liked and entered into negotiations with a salesman for the purchase of the car. The college student and the salesman negotiated the terms and financing for the sale of the car, including monthly payments, but the salesman informed the college student that he would not sell the car to the student unless the student had someone who could guarantee the loan that would be provided to purchase the car. The college student contacted his uncle who agreed to guarantee the loan for the student. The uncle accompanied the college student to the car dealership and met with the salesman. The uncle stated, "I'll guarantee the loan so that my nephew can buy this car and enjoy it." The salesman and the uncle shook hands. The salesman provided the college student with the loan necessary to purchase the car and the student took possession of the car. The college student subsequently failed to make any payments on the loan and the entire amount has become due. Can the salesman recover the entire amount of the loan from the uncle? Answers: No, because the agreement to guarantee the loan was not in writing. No, because there was no consideration for the uncle's promise to pay the debt. Yes, because the uncle agreed to guarantee the loan. Yes, because the uncle is primarily liable on the loan.

Answer choice A is correct. The Statute of Frauds applies to suretyship agreements. Suretyship is a three-party contract, wherein one party (the surety) promises a second party (the obligee) that the surety will be responsible for any debt of a third party (the principal) resulting from the principal's failure to pay as agreed. A promise to answer for the debt of another must generally be in writing to be enforceable. Here, the uncle agreed to act as a surety for the college student. However, the agreement was not in writing. Therefore, because the uncle's main purpose was not his own economic advantage and this is not an indemnity contract, the salesman cannot enforce the agreement against the uncle without a writing. Answer choice B is incorrect because the uncle did not have to receive a benefit for there to be consideration under the contract. The salesman merely had to have a detriment. Therefore, the contract is not unenforceable due to lack of consideration. Answer choice C is incorrect. Although the uncle agreed to guarantee the loan to the college student, the agreement was not enforceable because it was not in writing. Answer choice D is incorrect. The uncle was secondarily liable on the loan as a surety—he would only have to pay if the college student failed to pay. Moreover, even if the student never pays, the salesman cannot recover the amount of the loan from the uncle because the agreement cannot be enforced against him without a writing.

Question 6361 A car collector entered into an agreement with a car restorer to completely restore an exotic convertible. The car collector sought substantial work on the interior and exterior of the car, as well as the engine. The car restorer began working on the interior of the car and was able to complete a substantial portion of the interior restoration in one day. After the car restorer left his shop for the day, a severe storm occurred that contained highly damaging winds. A large tree located next to the shop was uprooted and landed directly on the exotic convertible, completely destroying it. The car restorer sought to recover the value of the work performed on the car's interior from the car collector. Will the car restorer's claim to recover the value of the work performed succeed? Answers: No, because no benefit was conferred by the car restorer upon the car collector. No, because the duties of both parties to the contract were discharged by impossibility. Yes, because the car collector's duty under the contract is not discharged. Yes, because the car restorer can recover the value of his service in quasi-contract.

Answer choice A is correct. The defense of impracticability (impossibility) is available if the specific subject matter of the contract is destroyed. In this case, the exotic convertible was destroyed through no fault of either party. As a result, the parties were no longer required to perform under the contract. The car restorer can still recover in quasi-contract for any benefit that was conferred prior to impracticability, but the court would only award restitutionary recovery in order to prevent the unjust enrichment. When a defendant is unjustly enriched by the plaintiff, restitution generally allows the plaintiff to recover on the benefit conferred by the plaintiff upon the defendant, rather than on the harm suffered by the plaintiff. Here, no benefit has been conferred, and thus the car restorer cannot recover the value of his service. Answer choice B is incorrect. Although the duties of both parties were discharged, this fact alone would not bar the car restorer's recovery in quasi-contract for the work performed. Therefore, answer choice A is the better answer. Answer choice C is incorrect because the duties of both parties are discharged due to impracticability. Answer choice D is incorrect. The fact that the exotic convertible was destroyed does not preclude the car restorer from recovering the value of the work performed prior to the convertible's destruction, but because no benefit was actually conferred upon the car collector, the car restorer cannot recover the value of his work.

Question 6396 A fashion designer who created clothing primarily for active wear decided to branch out into designing and selling sneakers for women. She contacted a sneaker manufacturer to make them for her. After negotiating the price and other terms, the two parties entered into a written contract whereby the manufacturer was to make the sneakers to the exact specifications of the designer. The manufacturer was to deliver the sneakers by May 15 on the condition that the designer sent the style specifications to the manufacturer by February 15. One day before the style specifications were due to the manufacturer, the designer was invited to showcase some of her clothing on a popular women's talk show. The date that the designer was to be on the show was February 17. The designer accepted the invitation, and spent the next three days working to prepare for her appearance on the talk show. As a result, she was unable send the style specifications for the sneakers to the manufacturer until February 27. The designer did not notify the manufacturer of the delay, and simply submitted the style specifications late. As a result, the manufacturer did not deliver the sneakers to the designer until June 1. Of the statements below, which accurately describes the legal obligations of the two parties at this time? Answers: The manufacturer was in breach of the contract because the sneakers were not delivered to the designer by May 15. The designer's failure to submit the style specifications on time constituted a waiver of the express condition of delivery by May 15. There was an implied condition in the contract that the designer give the manufacturer prompt notice of a delay in submitting the sneaker style specifications. The designer's twelve-day delay in submitting the sneaker style specifications excused the manufacturer's contractual duty to deliver the sneakers on May 15.

Answer choice A is correct. The general rule is that a party whose duty is subject to a condition can waive the condition, either by words or conduct. Here, the manufacturer's promise to deliver the sneakers by May 15 was subject to the condition that the designer provide the sneaker style specifications by February 15. Because the designer failed to provide the specifications by February 15, the manufacturer had the option to not perform under the contract or to waive the condition. The manufacturer chose to waive the condition by manufacturing the sneakers after receiving the style specifications on February 27. As a result of its waiver, the manufacturer was obligated to perform as promised under the contract and deliver the sneakers to the designer by May 15. Thus, the manufacturer breached the contract with the designer by delivering the sneakers on June 1. Answer choice B is incorrect. Delivery of the sneakers by May 15 was not an express condition of the contract and thus the designer could not waive it. Rather, the manufacturer's promise to deliver the sneakers by May 15 was subject to an express condition in the contract—that the designer provide the style specifications by February 15. After waiving the condition that the designer submit the specifications by February 15, the manufacturer had a duty to deliver the sneakers by May 15. This duty was based on a promise in the contract; it was not a condition. Accordingly, answer choice B is an incorrect statement of the parties' legal obligations. Answer choice C is incorrect. A court might imply a condition that the designer should give the manufacturer notice of a delay in submitting the specifications, but a court would be more likely to hold that the manufacturer waived the condition and thereby thought the 12-day delay was acceptable. Answer choice D is incorrect. The designer's failure to provide the style specifications by February 15 resulted in the failure of the express condition in the contract and excused the manufacturer's performance under the contract at that time. However, by accepting the specifications on February 27 and manufacturing the sneakers, the manufacturer waived the condition. Accordingly, the manufacturer had a contractual duty to deliver the sneakers on May 15.

Question 6135 A woman sent an offer to sell her office printer to her friend for $450. In her offer, the woman mentioned that an acceptance could be mailed to her business address, and that the friend should let her know within the next couple of weeks whether she was interested. The friend needed an office printer, so she immediately accepted the woman's offer by mailing a letter to the woman's home address. The woman only checked her mailbox at home once a week because she received so much junk mail, so she did not see the acceptance letter. Thinking that her friend was not interested, the woman sold her office printer to a different person. The next day, the friend came to the woman's house with a check for $450. The woman told the friend she had already sold the office printer. Will the friend succeed in an action for breach of contract? Answers: Yes, because the offer was irrevocable for at least two weeks. Yes, because the woman did not specify that mailing an acceptance to her business address was the only mode of acceptance. No, because it was not reasonable for the friend to mail her acceptance to the woman at the woman's home address. No, because the woman did not see the friend's acceptance letter.

Answer choice B is correct. The offeror can dictate the manner and means by which an offer may be accepted. Unless the offeror specifically requires the offeree to accept in a particular manner or by using a particular means, the offeree can accept in any reasonable manner and by any reasonable means. Here, the woman did not specify that the acceptance could only be mailed to her business address; she merely suggested that an acceptance could be mailed to her business address. Thus, another reasonable mode of acceptance, such as mailing it to the woman's home address, is permissible. Accordingly, answer choice C is incorrect. Answer choice A is incorrect because offerors may generally revoke offers at any time prior to acceptance, even if the offeror claims that the offer will be held open. Here, the friend would not succeed because the offer was irrevocable, but because she sent her acceptance before the offer was revoked. Answer choice D is incorrect. Under the mailbox rule, an acceptance is valid upon posting, not receipt. Here, the friend's acceptance was effective when she mailed the letter to the woman's home. Upon mailing the letter, an enforceable contract was created between the woman and the friend, even though the woman did not see the friend's acceptance letter.

Question 6142 A comic-book collector entered into a written contract with a man who had just inherited a large collection of comic books. They agreed that the man would sell to the collector numerous rare comic books for $4,000. Before they signed the agreement, the collector e-mailed the man to ask if he would include a particularly rare issue of Batman in the sale. The man agreed by e-mail that he would. The contract contained a list of each comic book that was supposed to be part of the sale, but the list did not include the Batman issue. The contract also stated that it was "the complete and final agreement" between the man and the collector. When the collector received the comic books, he discovered that the shipment did not include the Batman comic book. He sued the man, stating that even though the written contract did not mention the particular Batman issue, the collector had relied on the promise of the issue when he signed the contract. At trial, the collector seeks to introduce the e-mail he sent to the man prior to the execution of the agreement that referenced the Batman comic book. Is the e-mail admissible? Answers: No, because of the parol-evidence rule. No, because the Uniform Commercial Code does not apply to this transaction. Yes, because the agreement was only partially integrated. Yes, because the parol-evidence rule applies only to oral communications.

Answer choice A is correct. The parol-evidence rule generally prevents a party to a written contract from presenting prior extrinsic evidence that contradicts the terms of the contract as written. The rule is concerned with whether any of the earlier oral or written terms are part of the parties' contract, even though they are absent from the parties' written agreement. Generally, the parol-evidence rule prevents a party to a written contract from presenting other evidence that contradicts the terms of the contract as written. If a document is determined not to be "integrated," then the parol-evidence rule may not apply. When documents are only partially integrated, the parties are permitted to present extrinsic evidence as long as the evidence is consistent with the writing. The UCC (applicable to this contract) in essence presumes that a contract is a partial integration. However, if the writing completely expresses all of the terms of the parties' agreement, then it is a total integration, and the parties cannot introduce any extrinsic evidence (oral or written) of prior or contemporaneous understandings or negotiations. Here, the contract stated that it was the complete and final agreement and included a list of each comic book included in the sale, but it did not list the Batman issue in the list. Thus, the writing was a total integration, and the e-mail is inadmissible as extrinsic evidence of the agreement to include the Batman issue. Answer choice B is incorrect because the Uniform Commercial Code (UCC) does apply to this contract because it involves the sale of goods. Regardless, even if the UCC did not apply, the parol-evidence rule would still be applicable to the contract. Answer choice C is incorrect because the writing was a total integration, but even in a partially integrated agreement, the parol evidence may not contradict the writing. Answer choice D is incorrect because the parol-evidence rule applies to any previous or contemporaneous written or oral evidence prior to integration, not just oral communications.

Question 7094 A wheat farmer contacted an agricultural services company in May to inquire about hiring workers for a five-day period toward the beginning of the summer-long harvest season to assist the farmer in harvesting his wheat crop. After some negotiations, the farmer entered into a written contract with the company to provide five workers for a five-day period starting in the first week of June for a cost of $5,000. On June 5, the company's workers went on strike. On June 9, the strike ended and the company's workers began harvesting wheat on the farmer's farm, and did so for five days. The farmer subsequently refused to pay the company, claiming the company's delay in performance excused his obligation to pay. Is the farmer's obligation to pay excused? Answers: No, because the contract did not contain a "time is of the essence" clause. Yes, because substantially performing does not excuse a breach for commercial contracts. Yes, because starting in the first week of June was an express condition of the contract. Yes, because the delay was a material breach as the harvesting season had already begun.

Answer choice A is correct. Under common law, a material breach of contract allows the nonbreaching party to withhold any promised performance and to pursue remedies for the breach, including damages. A breach is considered minor when the breaching party has substantially performed. Here, both parties agreed time was not of the essence (because the completion date was not an express condition, as explained more fully below) and the delay does not appear to have been consequential (a somewhat nebulous start date combined with a summer-long harvesting season further supports this conclusion). For this reason, answer choice D is incorrect. Answer choice B is incorrect because it misstates the law, as there is no such rule that substantial performance does not apply to commercial contracts. Substantial performance does not apply to the sale of goods under the UCC, but does apply to contracts for services, such as the contract here. Answer choice C is incorrect because the "first week of June" start date was not an express condition. Express conditions generally contain language such as, "on condition that" or "provided that."

Question 7157 A caterer contracted with a local farmer for the delivery of three dozen fresh local eggs. The contract provided that because the caterer used the sterilized egg shells to serve one of her signature dessert recipes, the eggs needed to be a uniform color. The farmer delivered the caterer 20 white eggs and 16 speckled eggs. The caterer immediately e-mailed the farmer and informed him that she was rejecting the eggs because she could not use the inconsistent shells to serve her desserts. She also told him she could either sell them for him or save them for him to retrieve, and that she would wait for his instructions. The caterer saved the eggs for a week while she waited for the farmer's instructions, but he never responded. Finally, concerned that the eggs would spoil soon, she gave them to a friend. If the farmer brings a contract claim against the caterer for the contract price of the eggs she gave to her friend, will he succeed? Answers: No, because the caterer did not accept the eggs. No, because the caterer had no obligations regarding the nonconforming eggs. Yes, because the caterer had a duty to retain the eggs until the farmer retrieved them. Yes, because the caterer was required to sell the eggs on the farmer's behalf.

Answer choice A is correct. Under the UCC, if either the tender or the goods are nonconforming, then the buyer has the right to accept or reject all or part of the goods. Upon tender of nonconforming goods, a buyer must retain possession of the rejected goods for a reasonable period of time to allow the seller to reclaim them. However, a buyer's attempts in good faith to dispose of defective goods when the seller fails to give instructions within a reasonable time are not to be regarded as an acceptance. Here, the caterer properly rejected the eggs and tried to keep them until the farmer told her what to do with them, but she never received a response. Therefore, she did not accept the eggs, and the farmer cannot recover the contract price in a contract action. Answer choice B is incorrect because the caterer was obligated to notify the farmer that she was rejecting the eggs and to retain the eggs for a reasonable period of time for the farmer to reclaim them, which she properly did. Answer choice C is incorrect because the caterer was only required to retain the eggs for a reasonable amount of time. Based on the fact that she was concerned the eggs would spoil, she had already waited a reasonable amount of time with no response from the farmer when she gave the eggs to her friend. Answer choice D is incorrect. If rejected goods are perishable and the seller has no local agent to whom they can be returned, in the absence of other instructions from the seller, a merchant buyer is required to sell the goods on the seller's behalf. However, a non-merchant buyer is not required to sell the goods on the seller's behalf. Because the caterer is not in the business of selling eggs, she had no duty to sell these perishable goods on the farmer's behalf.

Question 6369 A private high school was in the market for new desks and chairs for its classrooms. It had inquired into the cost of acquiring 1,000 new desks and chairs from a particular vendor. On June 15, the vendor sent a signed letter to the private high school offering to sell 1,000 desks and chairs for $30,000. The letter stated that an acceptance of the offer by the high school would be effective so long as it was received by June 25. On June 23, the private high school mailed its signed, written acceptance of the vendor's offer. On June 24, the vendor, after concluding that the price it originally requested was too low, directly notified the private high school that it was revoking its offer. The vendor received the private high school's acceptance on June 26. Was an enforceable contract between the private high school and the vendor formed? Answers: No, because the private high school's acceptance was received on June 26. No, because the vendor revoked the offer on June 24. Yes, because the private high school mailed its acceptance on June 23. Yes, because the vendor's offer was irrevocable.

Answer choice A is correct. Under the mailbox rule, an acceptance that is mailed within the allotted response time is effective when sent (not upon receipt), unless the offer provides otherwise. Here, the vendor's offer stated that an acceptance would only be effective if it was received by June 25. Thus, the mailbox rule does not apply. The vendor received the private high school's acceptance on June 26. The acceptance is ineffective because it was not received by June 25. Therefore, the offer was no longer available when the acceptance was received. As a result, no contract was formed. Answer choice B is incorrect. Under the UCC, an offer to buy or sell goods is irrevocable if the offeror is a merchant, there is assurance that the offer is to remain open, and the assurance is contained in a signed writing by the merchant. Here, the vendor was a merchant and he informed the private high school in a signed letter that it could accept his offer by June 25. Accordingly, the vendor's offer was irrevocable until June 25. Thus, the vendor's attempted revocation of the offer on June 24 was ineffective. Answer choice C is incorrect because the acceptance had to be received by June 25—it was not effective upon mailing. Answer choice D is incorrect. Although the vendor's offer was irrevocable under the UCC firm offer rule, the offer was only irrevocable until June 25. The offer was no longer available on June 26 when the vendor received the private high school's acceptance.

Question 6403 A restaurant placed an order for 100 pounds of USDA prime beef from a meat packer. The order was to be shipped to the restaurant immediately. The meat packer erroneously shipped 100 pounds of USDA choice, rather than prime beef. When the choice beef arrived at the restaurant, which of the following is an accurate description of the restaurant's legal obligations? Answers: When the meat packer shipped the beef, a contract was formed, but the restaurant is not required to accept the choice beef. The shipment constitutes a counteroffer, which the restaurant can accept or reject. Since the shipment does not mirror the terms of the offer, there is no contract. The choice beef cannot be rejected because a contract was formed when the meat packer shipped the beef.

Answer choice A is correct. Unless an offer specifies otherwise, the offer may be accepted by either a promise or by performance. If the buyer requests that the goods be shipped, then the buyer's request will be construed as inviting acceptance by the seller either by a promise to ship or by prompt shipment of conforming or nonconforming goods. Here, the meat packer, by shipping the beef, accepted the offer. However, because the beef shipped did not conform to the terms of the offer, the shipment of choice rather than prime beef constitutes a breach of the contract. Consequently, the restaurant may accept the shipment of choice beef or reject it. Answer choice B is incorrect because the UCC generally treats the shipment of nonconforming goods as both an acceptance of the offer and a breach of the contract. Answer choice C is incorrect because it states the mirror-image rule for an acceptance under the common law, but the UCC controls in this case. Answer choice D is incorrect. Although a contract has been formed, the restaurant is still entitled to reject the beef as nonconforming.

Question 7026 On April 1, a buyer and a seller contracted in writing for the sale of an antique car for $20,000. The parties met on April 15, the scheduled date of the sale under the contract, and exchanged the car and a check. The check bore the following conspicuous notation: "This check is in full and final satisfaction of your payment under our April 1 contract." When the seller got to the bank to deposit the buyer's payment, the seller noticed the notation and also realized that the check was only for $15,000. Needing the money, the seller deposited the check anyway. The seller then sued the buyer for the $5,000 difference between the amount paid and the contract price. If the buyer's only defense is accord and satisfaction, is the buyer likely to succeed? Answers: No, because the amount the buyer owed the seller was liquidated and undisputed. No, because there was no consideration provided by the buyer to modify the contract. Yes, because the notation on the check formed a substitute contract. Yes, because the seller cashed the check knowing it was in full and final satisfaction of the contract.

Answer choice A is correct. When a party agrees to accept a lesser amount in full satisfaction of its monetary claim, there must be consideration or a consideration substitute for the party's promise to accept the lesser amount. For example, consideration can exist if the other party honestly disputes the claim or agrees to forego an asserted defense, or if the payment is of a different type than called for under the original contract . Here, there is no dispute over how much the buyer owed the seller. Therefore, the original debt cannot be discharged by this payment. Answer choice B is incorrect. Under the UCC, no consideration is necessary to modify a contract. Moreover, the buyer, in his notation on the check, attempted to create an accord agreement, not a modification, which requires a dispute over the amount owed or the validity of the debt. Answer choice C is incorrect because the notation on the contract is insufficient to form a substitute contract. Whether an agreement is an accord or a substitute contract turns on how formal the agreement is; the less formal, the more likely it is an accord. This notation on the check was more likely to be seen as an attempted accord, not a substitute contract. Answer choice D is incorrect because the buyer's defense will not succeed just because the seller cashed the check. The debt is still liquidated and undisputed, and even if the seller initially accepts a lesser amount, the seller may sue the buyer for the remainder due on the contract.

Question 4280 A woman emailed her friend, stating that she would like to buy the friend's teacup collection when "times aren't so tough. I would pay $1,000 for them." The friend responded with an email agreeing to the deal. The women did not exchange money or the teacups and did not see each other until a year later. When they did see each other, the friend apologized for forgetting about their discussion and told the woman she would deliver the teacups the next weekend and would accept a check at that time. The woman said she did not remember the discussion but would pay $750 for the teacups. The friend responded, "Haven't we already discussed this? Sold." The next day, the friend turned the teacups over to the woman, who provided her with a check for $750. The friend immediately responded that she needed the check for the remaining $250. The woman kept the teacups. Is the woman liable for the remaining $250? Answers: No, because a contract had not been formed. No, because a contract was not formed until the day the women spoke in person. Yes, because the original contract was for $1,000. Yes, because she kept the teacups.

Answer choice B is correct. A statement is an offer only if the person to whom it is communicated could reasonably interpret it as an offer. It must express the present intent of a person to be legally bound to a contract. The primary test of whether a communication is an offer is based on the objective theory of contracts; i.e., whether an individual receiving the communication would believe that he could enter into an enforceable deal by manifesting agreement to it. Here, the woman's email at the very most reflected a potential interest in buying the teacups at a future, unknown date. It does not reflect a present intent to be legally bound to buy the teacups. On the day of the oral discussion, however, she expressed a present intent to buy the teacups, and her friend would reasonably believe they could enter into an enforceable deal on that day. Answer choice A is incorrect because on the day of the oral discussion, the women did form an agreement regarding the teacups. Answer choice C is incorrect because as indicated above, the original letter indicated merely a possible interest in buying the teacups at a later date but no present intent to be bound by such an agreement. Answer choice D is incorrect because she accepted the teacups based on the agreement entered into on the day of the oral discussion.

Question 7020 In June, a local chef learned of a new business that opened in the area. Hoping to attract the business as a new client, the chef sent the business an offer consisting of a catalog of menus available through his catering service and a form letter he sent to all new businesses in the area. The letter was signed by the chef and included the following language: "Welcome! I specialize in creating delicious meals with local and organic ingredients, and I would be honored to be your catering source for all your business, promotional, and personal needs! To welcome you to the community, I would like to offer you a 25% discount off of my catalog prices on any three-course meal order, for up to 100 people, submitted this calendar year. I hope to hear from you soon and I look forward to doing business with you!" No communication occurred between the parties until the end of November, when the business faxed an order form to the chef requesting a catered meal for 60 people at a promotional event for a 25% discount. The court has found that both parties are merchants with respect to this transaction. Did the business's faxed order create an enforceable contract at a 25% discount? Answers: Yes, because the signed promotional letter created a firm offer. Yes, because the chef had not revoked the offer before the end of the calendar year. No, because the form letter was only an invitation to deal. No, because the company's power of acceptance terminated after a reasonable period of time.

Answer choice B is correct. An offer may be accepted at any time before the offer is terminated. Here, the offer specified that the offer would terminate at the end of the calendar year. Because the business submitted its order before the end of the calendar year, and because the chef has taken no action to terminate the offer, the offer remained open at the time the business submitted its order. Answer choice A is incorrect. Under the UCC, an offer to buy or sell goods is irrevocable if the offeror is a merchant, there is an assurance that the offer is to remain open, and the assurance is contained in a signed writing from the offeror. However, the irrevocability of a firm offer cannot exceed three months unless the offeree gives consideration to validate it beyond the three-month period. Therefore, even if a firm offer was created here under the UCC, it was no longer irrevocable in November; the chef could have revoked the offer, but didn't. Answer choice C is incorrect because even if the chef sent the letter to many businesses, the letter is more than a mere advertisement or invitation to deal. Here, the letter noted the terms of the offer and when the offer would terminate. This is sufficient to create an offer that the business could accept in November. Answer choice D is incorrect because this is only a true statement of law for situations in which the offer does not specify a time limit for acceptance. Because this offer specifically noted that the discount would apply to any order made before the end of the calendar year, the power of acceptance continued until the end of the calendar year.

Question 4287 A homeowner entered into a contract with a landscaper. The contract specified that the homeowner would pay the landscaper $10,000 upon completion of a list of projects. The landscaper performed the work while the homeowner was away on vacation. When the landscaper sought payment, the homeowner refused, noting that a tree had not been trimmed as required by the contract. The landscaper responded that, since he would now have to forego other work in order to trim the tree, he would do it but only if the homeowner agreed to pay him a total of $10,500 for his services. The homeowner, desperate to have the work completed, agreed. Once the work was completed, however, the homeowner gave the landscaper a check for $10,000, and refused to pay more. The landscaper sued for breach of contract. Is the landscaper likely to succeed in his claim? Answers: No, because an enforceable contract cannot be renegotiated. No, because there was no consideration for the promise to pay $10,500 and no unanticipated circumstances arose. Yes, because there was a valid modification of the contract. Yes, because the landscaper suffered a detriment by foregoing other work.

Answer choice B is correct. At common law, a promise to perform a preexisting legal duty does not qualify as consideration because the promisee is already bound to perform. In this case, the landscaper had a preexisting legal duty to trim the tree, and thus there was no consideration to support the homeowner's promise to pay an additional $500. Answer choice A is incorrect because an enforceable contract may be renegotiated. Even when there is a preexisting legal duty, there will be consideration if the promisee gives something in addition to what is already owed or varies the preexisting duty. Answer choice C is incorrect because modification of a services contract must be supported by consideration. Or some circumstances that were not anticipated when the contract was made must have arisen, and modification is fair and equitable in light of those circumstances. Answer choice D is incorrect because the fact that the landscaper had to forego other work would not serve as consideration in this case because the landscaper was under a preexisting legal duty.

Question 1973 An engineer entered into a written contract with an owner to serve in the essential position of on-site supervisor for construction of an office building. The day after signing the contract, the engineer was injured while bicycling and was rendered physically incapable of performing as the on-site supervisor. The engineer offered to serve as an off-site consultant for the same pay as originally agreed to by the parties. Is the owner likely to prevail in an action against the engineer for damages resulting from his failure to perform under the contract? Answers: No, because the engineer offered a reasonable substitute by offering to serve as an off-site consultant. No, because the engineer's physical ability to perform as on-site supervisor was a basic assumption of the contract. Yes, because the engineer breached the contract by disappointing the owner's expectations. Yes, because the engineer's duty to perform was personal and absolute.

Answer choice B is correct. The engineer is not in breach. The incapacity of a particular person to perform his or her duty under a contract renders the performance impracticable and operates as an excuse for nonperformance. The parties contracted for the engineer to personally provide on-site services. Therefore, the engineer's continued ability to perform those services was a basic assumption of the contract, and his nonperformance is excused. Answer choice A is incorrect. The parties contracted for the engineer to perform on-site services. An offer by a party to perform less than what was contracted for, even if the substituted performance is reasonable, will not excuse a breach. More importantly, his nonperformance is excused for the reasons set forth above. Answer choice C is incorrect because the engineer is not in breach. Answer choice D is incorrect. Although the engineer's duty to perform was personal and definite, he is not in breach.

Question 6409 A buyer contracted with an owner of commercial property located in a strip mall to purchase the property for $750,000. The contract called for closing and delivery of possession to occur on March 1. At the time of contracting, the owner informed the buyer that the current tenants were wrongfully refusing to vacate the premises, and would not do so until March 31. The owner notified the current tenants, who ran a call center on the premises, that they would need to vacate the premises before April 1. Although the current tenants stopped operating the call center before April 1, they were not able to empty the space completely because they had attached numerous cubicles to the floor, and the cubicles occupied the entire space of the property. Shortly after entering the contract, the buyer, who unbeknownst to the owner, planned on using the property as a gymnastics studio, ordered gymnastic equipment that was to be delivered on March 2. Due to the delay, the buyer was forced to rent a storage unit for this equipment for $1,000. By April 1, the fair market value of the property had risen to $755,000. In addition, the monthly fair market rental value of the property was $3,000. If the buyer files an action against the owner for damages, what will she likely recover? Answers: Consequential damages of $1,000 in storage unit costs, and expectation damages of $3,000 for the fair rental value of the property for the month of March. Expectation damages of $3,000 for the fair rental value of the property for the month of March. Nothing, because the property increased in value during the month of March. Nothing, because the delay was attributable to the tenants, not the owner.

Answer choice B is correct. The owner breached the contract by failing to deliver possession of the property on March 1. As a result, the owner is liable to the buyer for damages that the buyer suffered because of the breach. The buyer was denied possession of the property for the month of March. Therefore, the buyer is entitled to the fair market rental value of the property for a month, which is $3,000. Answer choice A is incorrect. The owner was unaware of the buyer's intended use of the property. Therefore, the buyer is not entitled to consequential damages for the cost of storing the gymnastic equipment for the month. Nor would the buyer be entitled to the storage fee as incidental damages. The buyer had notice that the previous tenants were wrongfully refusing to vacate and that the property would not be vacant until March 31. In light of this, it wasn't commercially reasonable to schedule the delivery for March 2. Answer choice C is incorrect because the buyer would have paid the same price, and received the same benefit, regardless of the owner's breach, which makes the increased value of the property insufficient compensation for the loss the buyer suffered. Answer choice D is incorrect. Although the owner may seek damages from the tenants for wrongfully holding over, the owner is nevertheless liable to the buyer for his own failure to deliver the property to the buyer on March 1, as contractually promised.

Question 6518 A jeweler and a goldsmith signed a written agreement that provided as follows: "For $3,000, the goldsmith shall sell to the jeweler a size six gold ring setting that the jeweler shall select from only the goldsmith's white gold ring designs." The agreement did not address any other specific terms with regard to the business arrangement between the jeweler and the goldsmith. When the jeweler arrived to select a ring, he refused to select one, claiming that the goldsmith, immediately prior to the execution of the written agreement, had orally agreed to broaden his choices to also include rose gold ring designs. The jeweler claimed that the goldsmith had, at the same time, also orally agreed to include a set of earring settings, valued at $1,000, as an incentive for his continued business. The goldsmith refused to sell to the jeweler any of his rose gold ring designs or include the earring settings. If the jeweler sues the goldsmith for damages, how should the court handle the evidence of the alleged oral agreements? Answers: The court should admit the evidence as to both the earring settings and the option to choose a rose gold ring design. The court should admit the evidence as to the earring settings but not the option to choose a rose gold ring design. The court should admit the evidence as to the option to choose a rose gold ring design but not the promise to include the earring settings. The court should admit neither the option to choose a rose gold ring design nor the promise to include the earring settings.

Answer choice B is correct. The parol evidence rule generally prevents a party to a written contract from presenting prior extrinsic evidence that contradicts the terms of the contract as written. If the writing completely expresses all of the terms of the parties' agreement, then it is a total integration, and the parties cannot introduce any extrinsic evidence (oral or written) of prior or contemporaneous understandings or negotiations. If, on the other hand, the writing sets forth the parties' agreement about some terms, but not all terms, then it is a partial integration. The UCC essentially presumes that a contract is a partial integration. Here, the UCC applies, so the contract will be presumed to be a partial integration, allowing additional consistent terms unless they certainly would have been included. The agreement between the jeweler and the goldsmith set forth only the size, price, and type of gold for the gold ring setting; it did not set forth any other relevant terms to their business agreement. When a writing is a partial integration, the parties are permitted to introduce supplementary extrinsic evidence (oral or written) of other terms as long as the evidence is consistent with the writing, but not if the evidence contradicts the terms of the writing. Here, the evidence about the earring settings only supplements the written agreement. However, the evidence about the option to choose a rose gold ring design directly contradicts the written agreement, which explicitly states that the jeweler can choose only from the white gold ring designs. Therefore, only the evidence as to the earring settings should be admitted. Answer choices A and C are incorrect, as the evidence about the option to choose a rose gold ring design expressly contradicts the written agreement and thus cannot be admitted. Answer choice D is incorrect, as the evidence about the earring settings only supplements the written agreement and thus would be admissible under the parol evidence rule.

Question 1985 A lawn service company agreed in writing to purchase from a supplier all of its requirements for lawn care products during the next calendar year. In the writing, the supplier agreed to fulfill those requirements and to give the company a 10% discount off its published prices, but it reserved the right to increase the published prices during the year. After the parties had performed under the agreement for three months, the supplier notified the company that it would no longer give the company the 10% discount off the published prices. Does the company have a viable claim against the supplier for breach of contract? Answers: Yes, because part performance of the agreement by both parties made it enforceable for the full year. Yes, because the company's agreement to buy all of its lawn care products from the supplier made the agreement enforceable. No, because the supplier could, and did, revoke its offer with respect to future deliveries. No, because the absence of a minimum quantity term rendered the company's promise illusory.

Answer choice B is correct. The parties entered into an enforceable requirements contract under UCC § 2-306. Although the terms of the parties' agreement granted the supplier the discretion to increase the published prices during the year, the contract did not grant the supplier the right to discontinue the promised 10% discount off the published prices. The supplier's refusal to give the company the 10% discount was a breach of the contract's agreed-upon terms. Answer choice A is incorrect because, pursuant to the terms of their contract, the parties agreed to a one-year term. Therefore, part performance is not required in order for the contract to be enforceable for a full year. Answer choice C is incorrect because revocation relates to the contract formation process. Given that mutual assent has occurred, revocation is not a relevant issue. Answer choice D is incorrect. UCC § 2-306(1) validates the enforceability of requirements contracts even if the parties do not include a minimum quantity term in their agreement. Therefore, the absence of a minimum quantity term in the parties' requirements contract did not render it illusory, and the contract is enforceable.

Question 1955 A computer retail outlet contracted to service a bank's computer equipment for one year at a fixed monthly fee under a contract that was silent as to assignment or delegation by either party. Three months later, the retail outlet sold the service portion of its business to an experienced and well-financed computer service company. The only provision in the agreement between the retail outlet and the computer service company relating to the outlet's contract with the bank stated that the outlet "hereby assigns all of its computer service contracts to [the computer service company]." The computer service company performed the monthly maintenance required under the service contract. Its performance was defective, however, and caused damage to the bank's operations. Whom can the bank sue for damages arising from the computer service company's defective performance? Answers: The retail outlet only, because the computer service company made no promises to the bank. Either the retail outlet or the computer service company, because the bank has not released the outlet and the bank is an intended beneficiary of the outlet's agreement with the computer service company. Either the retail outlet or the computer service company, because since each has the right to enforce the bank's performance of its contract with the retail outlet, mutuality of remedy renders either potentially liable for the defective performance. The computer service company only, because it is a qualified and a financially responsible supplier of computer services.

Answer choice B is correct. The retail outlet effectively delegated to the computer service company the outlet's duty of performance owed to the bank. However, absent consent by the obligee (the bank) or performance by the delegate (the computer service company), an effective delegation does not relieve the delegating party (the retail outlet) of its duty to the obligee. When the computer service company purchased the service contracts, the bank also became an intended beneficiary of the computer service company's promise to the retail outlet. The computer service company expressly agreed with the retail outlet to perform the outlet's obligation to the bank, making the bank an intended beneficiary of that obligation. Thus, both the retail outlet and the computer service company remain liable to the bank. Answer choice A is incorrect because the computer service company expressly agreed with the retail outlet to perform the outlet's obligation to the bank, thus imposing a duty on the computer service company of which the bank was an intended beneficiary. Therefore, the bank is also entitled to sue the computer service company for its defective performance. Answer choice C is incorrect because, assuming an effective assignment, the right of the retail outlet to the bank's performance was extinguished. It is true that the bank can sue either the retail outlet or the computer service company, but it can do so (a) because the bank has not released the outlet and (b) because the bank is an intended beneficiary of the outlet's agreement with the computer service company. Answer choice D is incorrect because the competence and financial responsibility of the retail outlet's delegate, the computer service company, does not discharge the retail outlet's obligation to the bank.

Question 3186 A retailer received a written firm offer signed by a supplier. The offer committed the supplier to providing the retailer with up to 10,000 tubes of toothpaste over the next 45 days at $1 a tube. Thirty days later, the supplier informed the retailer that the price per tube of toothpaste would be $1.10. The next day the retailer ordered 6,000 tubes of toothpaste from the supplier, which the supplier promptly shipped. Sixty days after the receipt of the offer, the retailer ordered another 4,000 tubes of toothpaste, which the supplier also promptly shipped. What price is the supplier permitted to charge the retailer for the toothpaste? Answers: $10,000 (10,000 * $1), because the supplier's firm offer was effective for three months regardless of its terms. $10,400 ((6,000 * $1) + (4,000 * $1.10)), because the supplier's firm offer was effective for only 45 days. $11,000 (10,000 * $1.10), because the firm offer rule does not apply where the buyer is a merchant. $11,000 (10,000 * $1.10), because the supplier informed the retailer that the price was increased to $1.10 before the retailer's placement of either order.

Answer choice B is correct. The supplier's written and signed firm offer was effective and irrevocable for 45 days, even though no consideration was provided by the retailer to the supplier for making the firm offer. Consequently, the price for the first shipment of 6,000 tubes of toothpaste was $1 per tube. Since the firm offer had expired prior to the placement of the second order, the supplier's price of $1.10 per tube was in effect with regard to this order. Answer choice A is incorrect because, while a firm offer made by a merchant may be effective up to three months, the merchant offeror, as with any offeror, is master of the offer. Consequently, the merchant offeror could set the time period for the firm offer at 45 days. Answer choice C is incorrect because the firm offer rule applies, even where the buyer is a merchant, as long as the seller is a merchant. Answer choice D is incorrect because, as noted with regard to answer choice B, the firm offer made by the supplier applies to the first order placed by the retailer because such offer was irrevocable.

Question 1989 A wholesaler contracted in a signed writing to sell to a bakery 10,000 pounds of flour each week for 10 weeks, the flour to be delivered to the bakery on Mondays and payment to be made on Wednesdays of each week. The bakery did all of its weekly bread baking on Tuesdays. On Monday morning of the first week, the wholesaler tendered delivery of 8,000 pounds of flour to the bakery, and the bakery accepted it on the wholesaler's assurance that the remaining 2,000 pounds would be delivered later that evening, which it was. The bakery paid for both deliveries on Wednesday. On Monday of the second week, the wholesaler tendered delivery of 5,000 pounds of flour to the bakery and said that the remaining 5,000 pounds could not be delivered on Monday but would be delivered by Wednesday. The bakery rejected the tender. Was the bakery legally justified in rejecting the tender of the 5,000 pounds of flour? Answers: Yes, because the bakery was legally entitled to reject any tender that did not conform perfectly to the contract. Yes, because the tender was a substantial impairment of that installment and could not be cured. No, because the tender was not a substantial impairment of the entire contract, and the wholesaler had given assurance of a cure. No, because by accepting the first 8,000 pounds on Monday of the first week, the bakery had waived the condition of perfect tender and had not reinstated it.

Answer choice B is correct. UCC § 2-612(2) provides that a buyer may reject any installment that is nonconforming if the nonconformity substantially impairs the value of that installment and it cannot be cured. The delivery of less than the contracted-for amount constituted a nonconformity that substantially impaired the value of the installment since the wholesaler could not deliver the remaining 5,000 pounds until the day after the bakery needed the flour to fulfill its weekly baking needs, and the bakery was entitled to reject the tender. Answer choice A is incorrect because, although UCC Article 2 adopts the perfect tender rule, that rule is inapplicable to assess the conformance of a seller's performance under an installment contract. Because this is an installment contract, the substantial impairment standard governs the buyer's right to reject, and the bakery is entitled to reject under that standard. Answer choice C is incorrect. A buyer's right to reject a particular installment is not dependent on a substantial impairment of the entire contract. Moreover, in this case, the wholesaler's assurance of cure was inadequate since the wholesaler could not deliver the remaining 5,000 pounds until the day after the bakery needed the flour to fulfill its weekly baking needs. Answer choice D is incorrect because a buyer's previous acceptance of a nonconforming tender does not constitute a waiver of its right to reject a later nonconforming tender if the other grounds for rejection are satisfied.

Question 7033 A construction company contracted with a manufacturer to purchase 100 identical prefabricated windows to use while constructing houses in a gated community. The windows were to be delivered in shipments of 25 windows each on April 1, May 15, July 1, and August 15. The written contract, signed by both parties, was silent as to when payment for each shipment would be due. The manufacturer made the first two shipments in conformity with the contract requirements, and the construction company paid one-fourth of the full contract price upon each delivery. However, on June 1, the manufacturer demanded that the construction company pay the entire remainder of the contract price before the manufacturer would make any further shipments. Which of the following statements is true? Answers: The construction company has no duty under the contract to make any payments until the final delivery is made. The construction company must pay the manufacturer one-fourth of the contract price upon delivery of each conforming shipment of windows. The construction company's failure to pay the requested sum will amount to a repudiation of the contract. The manufacturer waived his right to demand immediate payment of the full contract price when he accepted the first payment of one-fourth the contract price on April 1.

Answer choice B is correct. Under the UCC, an installment contract is defined as one in which the goods are to be delivered in multiple shipments, and each shipment is to be separately accepted by the buyer. Payment by the buyer is due upon each delivery, unless the price cannot be apportioned. Therefore, the construction company is only obligated to pay the manufacturer an apportioned price for each conforming delivery (i.e., one-fourth of the full contract price). Answer choice A is incorrect because the price of these windows can be easily apportioned between the four shipments. Therefore, an apportioned payment is due upon each delivery. Answer choice C is incorrect because the construction company's denial of the manufacturer's demand for immediate payment of the remainder of the contract price will not amount to a clear and unequivocal refusal to perform under the contract. Therefore, this conduct would not operate as a repudiation. Answer choice D is incorrect. Under the UCC, payment for installment contracts is due upon each delivery, unless the price cannot be apportioned. Parties cannot vary or contract out of this definition under the code. Accordingly, the manufacturer did not have the right to demand immediate payment of the full contract price at any time.

Question 4371 A carpenter and a homeowner entered into a valid written contract for the carpenter to design, build, and install kitchen cabinets in the homeowner's house at a cost of $25,000; the price includes the cost of materials used, but mostly reflects the cost of the carpenter's services. After execution of the contract, but before beginning performance, the carpenter realized that his initial calculations were incorrect. The carpenter phoned the homeowner, explained the matter, and demanded an additional $5,000 in order to perform the job. The homeowner orally agreed, after securing the carpenter's consent to change the style of handles and hinges to be used on the cabinets, a change that, unaware to the homeowner, resulted in a cost savings to the carpenter. The carpenter constructed and installed the cabinets. When the homeowner tendered a check for $25,000 to the carpenter, the carpenter demanded the agreed-upon contract price of $30,000. In a common-law jurisdiction, is the carpenter likely to prevail in his demand for the full $30,000? Answers: Yes, because the homeowner agreed to pay the higher amount. Yes, because the carpenter's duties under the contract were modified. No, because the carpenter did not suffer a financial detriment. No, because the oral modification violated the Statute of Frauds.

Answer choice B is correct. Unlike a contract for goods, modification of a contract for services must be supported by consideration. Where a contract has both services and goods elements, the predominant purpose test applies. Since the cost of the carpenter's services exceed those of the materials used to make the cabinets, this contract is likely to be classified as a contract for services. Consideration can be found when there is a change in a party's duties, even where that change is financially beneficial to the party. Answer choice A is incorrect because, although the homeowner did agree to pay the higher amount, this promise is unenforceable under the pre-existing duty rule unless there is variation in the carpenter's duties. In this case, the carpenter agreed to use different hardware on the cabinets. Answer choice C is incorrect because, although the carpenter did not suffer a financial detriment as a result of the change in the type of hardware to be used on the cabinets, the carpenter's duties were varied. Generally, a court will not evaluate the adequacy of consideration where the consideration is given as part of a bargained-for exchange. Answer choice D is incorrect because, although the Statute of Frauds applies to a contract that, as modified, falls with its provisions, this contract, being one for services, does not violate the Statute of Frauds. Even if the value of the materials used by the carpenter is $500 or more, the UCC Statute of Frauds provision does not apply because the contract will be classified as one for services, not goods.

Question 6386 On May 1, a clothing manufacturer sent a written offer to a retailer for the sale of 1,000 pairs of designer jeans at a price of $50 per pair, including delivery costs. The proposed delivery date was June 15. On May 5, the retailer mailed a letter accepting the manufacturer's offer. On May 15, a natural disaster occurred causing fuel prices to significantly increase. As a result, the clothing manufacturer sent a letter to the retailer requesting an additional $500 to cover the increased delivery and transportation costs for the shipment of jeans. The retailer considered seeking out a new supplier, but hoped to continue doing business with this manufacturer in the future, so decided against it. The retailer returned a signed letter promising to pay the additional $500. On June 15, the clothing manufacturer delivered 1,000 pairs of jeans to the retailer. The retailer paid the clothing manufacturer $50,000, but refused to pay the additional $500. Can the manufacturer enforce the retailer's promise to pay an additional $500? Answers: Yes, because the manufacturer relied on the promise when it delivered the jeans to the retailer. Yes, because the manufacturer made its request for the additional $500 in good faith. No, because the attempted modification was unconscionable. No, because no consideration supported the promise to pay the additional $500.

Answer choice B is correct. Unlike under the common law, under Article 2, no consideration is necessary to modify a contract; however, good faith is required. Good faith requires honesty in fact and fair dealing in accordance with reasonable commercial standards. If a party demands an increase in price because the other party has no choice but to agree, the courts will invalidate such a bad-faith modification. Here, a natural disaster occurred causing fuel prices to significantly increase. As a result, the clothing manufacturer's request for an additional $500 to cover the increased delivery and transportation costs was made in good faith, based on the natural disaster. Also, the retailer was not forced into the agreement because other suppliers were available. Therefore, the attempted modification was valid, and the manufacturer can enforce the retailer's promise to pay an additional $500 without reliance or additional consideration. Answer choice A is incorrect. Promissory estoppel is not necessary here because there was an enforceable modification of the contract. Thus, the manufacturer can enforce the promise under the valid contract and need not show reliance. Answer choice C is incorrect because the request for an additional $500 does not make the modified contract unconscionable. A contract (or part of a contract) is only unconscionable when it is so unfair to one party that no reasonable person in the position of the parties would have agreed to it. Answer choice D is incorrect because contracts under the UCC, such as the one here, do not require consideration for modification; rather, only good faith is required.

Question 6347 A man and a woman entered into a valid contract with each other to provide transportation services for their neighbor, who recently had major surgery on her hip. The contract provided that the man and woman would each pay $100 each month for a period of one year to a local black car service. The man and woman selected the particular black car service based on recommendations that indicated it was very professional and always punctual. After one month of performance under the contract, the man refused to pay the black car service under the contract and the black car service stopped providing service to the neighbor. The woman brought an action seeking specific performance by the man pursuant to the terms of the contract. Will the woman be successful? Answers: No, because contracts for personal services cannot be enforced by specific performance. No, because monetary damages are sufficient to compensate the woman. Yes, because her damages cannot be calculated with reasonable certainty. Yes, because monetary damages are an inadequate remedy.

Answer choice B is correct. When damages are an inadequate remedy, the nonbreaching party may pursue the equitable remedy of specific performance. In determining whether the legal remedy is adequate, the court will consider a variety of factors, including the difficulty of proving damages with reasonable certainty, hardship to the defendant, balance of the equities, the wishes and understandings of the parties practicality of enforcement, and mutuality of the agreement. Here, the man and woman entered into a valid contract requiring the payment of $100 per month for a period of one year. Accordingly, the woman's damages can be calculated with reasonable certainty and are an adequate remedy in this instance. Answer choice A is incorrect. Courts rarely grant specific enforcement of contracts for personal services. Although the contract was made for the provision of transportation services, the contract did not require the personal services of either the man or the woman. Therefore, specific performance would have been an available remedy if damages were inadequate. Answer choice C is incorrect. The woman's damages can be calculated with reasonable certainty because the contract called for the payment of $100 by the man for a period of one year. Answer choice D is incorrect because damages are an adequate remedy for the man's breach in this case.

Question 6013 A mechanic and a farmer contracted in writing for the repair of the farmer's tractor, with a payment of $2,000 due upon completion. The mechanic called the farmer on April 15 to inform the farmer that the work was complete. When the farmer went to pick up the tractor the next day, he told the mechanic that due to an unforeseen rise in feed costs, he couldn't pay the full contract price. The farmer paid the mechanic the first $1,000, and the mechanic told the farmer that, if the farmer promised to pay the remainder by June 1, then the mechanic would not sue to recover the remaining $1,000. The farmer orally agreed. On May 1, the mechanic sued the farmer for the unpaid $1,000, and the farmer filed a motion to dismiss. Should the court grant the motion to dismiss? Answers: No, because the new cost of feed is not an unforeseen difficulty that would allow for modification of the existing contract. No, because there is no consideration to support the mechanic's promise not to sue. Yes, because a promise to allow a debtor to delay payment on a past debt is enforceable without consideration. Yes, because the payment of $1,000 was sufficient consideration to support the mechanic's promise not to sue.

Answer choice B is correct. While the law permits the settlement of debts, consideration is required for a settlement to be enforceable. Under the preexisting duty rule, the mechanic's promise to forbear from suing to collect was not supported by consideration from the farmer, because the amount due was liquidated and the farmer did not promise to do anything more than he was already obligated to do. The farmer did pay $1,000, but as a partial payment of an undisputed debt, that money is insufficient consideration for the mechanic's promise not to sue. Answer choice A is incorrect because impracticality would only apply as a defense to the original repair contract, not the forbearance agreement. Answer choice C is incorrect because a promise to allow a debtor to delay payment on a past debt is not enforceable without consideration. Answer choice D is incorrect because the payment of $1,000 of the $2,000 owed was a partial payment of an undisputed debt, and cannot serve as consideration for the mechanic's agreement not to sue the farmer.

Question 7016 On January 5, a buyer and seller contracted for the delivery of 100 widgets if they could be delivered by February 20. The agreement was made in a writing signed by both parties and provided that the buyer would pay the contract price of $1,000 upon delivery. On February 3, the buyer and seller orally agreed to postpone delivery until March 1. However, when the widgets arrived on March 1, the buyer refused to accept or pay for the widgets. If the seller sues the buyer for breach of contract, who is most likely to succeed in the action? Answers: The buyer, because any modification of the parties' contract must satisfy the Statute of Frauds. The buyer, because the agreement on February 3 was not supported by consideration. The seller, because the oral agreement on February 3 waived the February 20 delivery date. The seller, because the contract modification on February 3 was immediately binding on both parties.

Answer choice C is correct. A party whose duty is subject to the condition can waive the condition, either by words or by conduct. A party who indicates that a condition will not be enforced may be estopped from using that condition as a defense if the other party reasonably relied on the party's words or conduct that the condition had been waived. Here, the buyer and the seller agreed to postpone delivery (i.e., the buyer waived the condition that the widgets be delivered by February 20). The seller relied on that waiver (as evidenced by the fact that he did not deliver the widgets until March 1), and it would be unjust for the buyer to now claim that the contract was breached. Therefore, the seller will prevail because the February 20 delivery date was waived. Answer choice A is incorrect. A promise that the promisor should reasonably expect to induce action or forbearance on the part of the promisee or a third person and that does induce the action or forbearance is enforceable notwithstanding the Statute of Frauds if injustice can be avoided only by enforcement of the promise. Here, the buyer's indication that delivery would be accepted on March 1 prevents the buyer from using the seller's failure to deliver by the February 20 delivery deadline to justify the buyer's failure to perform under the contract. Answer choice B is incorrect because consideration is not needed to modify a contract under the UCC. Answer choice D is incorrect because contract modification (i.e., the waiver) was not technically enforceable until there had been detrimental reliance. The requirements of the Statute of Frauds must be satisfied if the contract as modified is within its provisions and as a contract for the sale of goods over $500, such a modification would require a writing. However, once the buyer let the original delivery date pass, the buyer was estopped from using the passing of that deadline as justification for failing to perform.

Question 6402 An amateur bodybuilder entered into a contract with a personal trainer to help the bodybuilder prepare for his first bodybuilding competition. The written agreement stated that the bodybuilder would pay the personal trainer $4,000 to do daily weight-lifting sessions, and teach him about competition musculature and proper nutrition. Per the contract, the training would occur over a period of eight weeks leading up to the competition, and the bodybuilder would pay the personal trainer at the end of the eight weeks. Three weeks before the competition, the bodybuilder was lifting weights with the guidance of the personal trainer, when suddenly a beam fell from the gym's ceiling and onto the bodybuilder while he was lifting weights. The bodybuilder suffered severe head trauma, and was put into a medically induced coma for more than a month. With regard to the contractual duties of each party, which of the following is a correct statement? Answers: Only the personal trainer's duty to perform is dismissed, but he can recover reliance damages from the bodybuilder for the weeks that he did train the bodybuilder. Each party's duty to perform is dismissed, and the personal trainer cannot recover anything from the bodybuilder. Each party's duty to perform is dismissed, but the personal trainer can recover under a theory of quasi-contract for the weeks that he did train the bodybuilder. Only the personal trainer's duty to perform is dismissed, but the bodybuilder is still obligated under the contract because the amount that he owed the personal trainer can be determined.

Answer choice C is correct. A party's duty to perform can be discharged by impracticability. Here, the parties executed a valid contract, but performance on both sides became impracticable because of the occurrence of an unforeseeable event (the falling beam injuring the bodybuilder), the nonoccurrence of which was a basic assumption of the contract. At that point, both parties' contractual duties ceased—the trainer did not have to provide training services, and the bodybuilder did not have to pay the $4,000 due under the contract. Although there would be no legal recovery, the equitable doctrine of restitution (quasi-contract) would apply to prevent unjust enrichment. A party may recover in quasi-contract for any benefit that was conferred prior to impracticability. In this case, the bodybuilder would have to pay the trainer $2,500 for the services already rendered (i.e., the five weeks of training). Answer choice A is incorrect because both parties' duties are discharged, not merely the personal trainer's. Answer choice B is incorrect, as quasi-contractual recovery is permitted for any benefit conferred prior to impracticability. Answer choice D is incorrect because both parties' duties are discharged, not merely the personal trainer's.

Question 6132 At a local market, a buyer offered to purchase a large, framed mirror from an artist for $1,000. The artist stated that he wanted to wait to see how many people went through the market that day before he decided on whether he would accept the offer. The next morning, the buyer returned to the market only to learn that the mirror had been dropped and broken. Because the frame of the mirror was still in good condition, the buyer wrote a check for $500 and gave it to the artist without further remark. The artist loaded the frame into the buyer's vehicle and demanded the remaining $500 offered the day before. Is the buyer liable for the remaining $500? Answers: Yes, because the original offer was still valid. Yes, because the artist thought that he had accepted the original offer. No, because the original offer terminated. No, because the buyer reasonably believed the original offer terminated.

Answer choice C is correct. An offer involving subject matter that is destroyed is terminated. In this case, when the buyer originally offered $1,000, the subject of the offer was a large, framed mirror. When the mirror broke, the buyer's offer terminated. The artist could no longer accept it. Answer choice A is incorrect because the original offer terminated upon the destruction of the mirror. Answer choice B is incorrect because the offer terminated upon the destruction of the mirror even if the artist believed otherwise. Further, the artist's subjective belief is irrelevant, as contract law takes an objective approach in determining intent. Answer choice D is incorrect because regardless of the buyer's belief, the offer had terminated.

Question 2802 An honest dispute develops between a condominium owner and a plumber over whether plumbing installed in the kitchen and bathrooms of the condominium satisfied contractual specifications. If the plumbing meets those specifications, the condominium owner owes the plumber $15,000 under the terms of the contract. The condominium owner offers to pay the plumber $10,000 in satisfaction of the owner's contractual obligations, if the plumber replaces the plumbing in the kitchen with another grade of pipe. The plumber accepts the condominium owner's offer. After the plumber replaces the kitchen plumbing, the condominium owner refuses to pay the plumber. In a breach of contract action brought by the plumber, the fact-finder determines that the plumbing originally installed by the plumber did satisfy the contract specifications. The fact-finder also determines that the plumber and the condominium owner entered into a substitute agreement for which the owner failed to deliver the required performance. What is maximum amount that the plumber can seek in damages from the condominium owner? Answers: $25,000 $15,000 $10,000 Nothing

Answer choice C is correct. Because the plumber and the condominium owner entered into a substitute agreement rather than an accord, the substitute agreement completely replaces the original contract. Consequently, the plumber may only seek damages of $10,000, the amount that the condominium owner promised to pay under that agreement. Had the second agreement between condominium owner and the plumber been an accord, the plumber could have sought damages upon the condominium owner's breach of the accord under either the original contract (i.e., $15,000) or the accord (i.e., $10,000), but not both. For this reason, answer choices A, B, and D are incorrect. NOTE: Be careful with this question. Even though the facts tend to read like an accord, we know that this is instead a substitute agreement because the question states that the fact-finder determined this to be so.

Question 7022 A library contacted a local artist expressing an interest in purchasing a particular one of the artist's sculptures for display at the library. The library's agent and the artist executed a written contract signed by both parties providing that the library would purchase the sculpture for $1,000 due upon delivery of the sculpture to the library. Just before they signed the contract, the agent told the artist, "Plan on delivering the sculpture in 10 days, but please remember that the library's obligation to purchase the sculpture will be conditioned on the approval of the chairperson of the Artistic Patronage Council, as they will be providing the library with the funds for this sale." The chairperson of the Artistic Patronage Council orally approved the sale the next day. However, ten days after the contract was executed, the artist decided he did not want to sell the sculpture. If the library sues the artist for breach of contract, is the library likely to prevail? Answers: No, because the library's agent made an illusory promise. No, because there was no mutuality of remedy when the contract was executed. Yes, because the agreement was supported by good consideration even though it was conditioned on an uncertain event. Yes, because the artist waived any lack of consideration by signing the contract.

Answer choice C is correct. Consideration can take the form of a return promise to do something, a return promise to refrain from doing something legally permitted, the actual performance of some act, or refraining from doing some act. Consideration does not fail simply because a party's performance is conditioned on an uncertain event, such as approval by a third party. Therefore, the library can succeed in a breach of contract action against the artist. Answer choice A is incorrect. An illusory promise is one that essentially pledges nothing because it is vague or because the promisor can choose whether or not to honor it. A promise that is based on the occurrence of a condition within the control of the promisor may be illusory, but there is no evidence on the facts that the approval of the chairperson of the Artistic Patronage Council is within the control of either the library or the library's agent. Therefore, this does not appear to be an illusory promise. Answer choice B is incorrect because mutuality of remedy is not a requirement to form a valid contract. Answer choice D is incorrect. The basic concept of consideration is that there must be something of substance given in exchange for the promise that is to be enforced. The court generally will not review the adequacy of the consideration supporting a contract, but the requirement for some form of consideration cannot be waived.

Question 6412 A public high school with over 800 students hosted a walkathon to raise money to purchase new textbooks for their students. The school administration solicited various businesses to sponsor the walkathon, including the CEO of a large food services company. The CEO wanted the administration to hire his food services company to provide lunch in the school cafeteria when the school's contract with their current food services provider expired in one month. Hoping to catch the administration's attention, the CEO pledged that his company would match all of the sponsors' pledges. Because he was a business sponsor, the CEO was required to fill out and sign a writing stating this promise. The walkathon was a success, and the total amount raised, excluding the CEO's pledge, was $42,000. After collecting the pledge money from all of the sponsors except for the CEO, the school was able to replace the old textbooks at a total cost of $42,000. The school planned on using the CEO's pledge to purchase supplemental learning materials to further advance the education of its students. Prior to being asked for his matching pledge, the CEO learned that the contract of the school's current food services provider had been renewed for another five years. The CEO subsequently repudiated his promise to match all of the sponsors' pledges. If the school sues the CEO for $42,000, will it succeed? Answers: No, because the school did not actually rely to its detriment on the CEO's pledge when purchasing the textbooks. No, because there is no substantial injustice since the school was able to pay for all of the textbooks without the CEO's pledge. Yes, because the court will presume that the school detrimentally relied upon the CEO's written promise. Yes, because the CEO acted in bad faith when he repudiated his promise to match the pledges of the other sponsors.

Answer choice C is correct. Courts often apply the doctrine of promissory estoppel to enforce charitable subscriptions. In some cases, they presume that the charity detrimentally relied on the promised contribution. A charitable subscription (i.e., a written promise) is enforceable under the doctrine of promissory estoppel without proof that the charity relied on the promise. Here, there is no proof in the facts that the school relied upon the CEO's promise, but that promise is a charitable subscription, the court will presume that the school detrimentally relied on the CEO's written promise. For this reason, answer choice A is incorrect. Answer choice B is incorrect because the CEO should have reasonably expected that the school would rely upon his pledge, and that there would be injustice if he did not pay the $42,000. Answer choice D is incorrect. Whether the CEO acted in bad faith is not the issue. Rather, the central issue here is that a court will enforce the written promise under the doctrine of promissory estoppel without proof that the school relied on the CEO's pledge.

Question 6374 A butcher and a seller entered into a written contract for the purchase and sale of a building to be used as a butchery. The closing was scheduled for June 1. On May 25, the seller was notified by the city that the building, which had previously been used as a butchery, had a number of significant city code violations. The seller immediately contracted with an electrician and others to correct the issues. Despite his best efforts, the seller realized the building would not be brought up to code until at least June 10. The seller promptly sent written notification of this issue to the butcher and informed him that he would be unable to take possession of the building until June 10. Based on his agreement with the seller, the butcher had declined to renew his lease at his current location and was forced to remove his equipment and inventory from his current location by the end of May. Between June 1 and June 10, to prevent spoilage of his inventory due to the delay, the butcher had to rent space to store his equipment and inventory. He moved his freezer to the rented space to store his meat at a cost of $200 per day, plus the cost of electricity to run the freezer. On June 10, the building was up to code. The butcher paid the seller the agreed-upon purchase price of $300,000 and took possession of the property. What damages, if any, may the butcher recover from the seller? Answers: Nothing, because the seller acted in good faith. Nothing, because the contract did not contain a "time is of the essence" clause. $2,000, the rental cost he had to pay from June 1 to June 10. $2,000 plus the cost for electricity to run the freezer.

Answer choice C is correct. Expectation (benefit-of-the-bargain) damages are intended to put the nonbreaching party in the same position as if the contract had been performed as agreed. Expectation damages must be calculated with reasonable certainty. If the seller breached but acted in good faith, then damages are limited to the buyer's out-of-pocket expenses. In this case, the butcher is entitled to be compensated for the loss that resulted from the seller's (minor) breach, which is the $2,000 in rental costs that the butcher had to pay from June 1 to June 10. Answer choice A is incorrect. The seller remains liable for breach even though he acted in good faith. However, the seller's good faith limits the butcher's damages to out-of-pocket expenses only. Answer choice B is incorrect because the absence of a "time is of the essence" clause in the contract does not prevent the butcher from recovering his out-of-pocket expenses from the seller. Answer choice D is incorrect because the butcher would have had to pay the same cost for electricity to run the freezer in the building he purchased if he had been able to move in.

Question 6133 Prior to her death, a celebrity commissioned an artist to paint a portrait of her. The celebrity hired this particular artist because he only painted using an old-fashioned and rarely used style that required two months of daily appointments during which the subject would sit for the painting over a few hours each day. The contract between the parties specified that this live-model method be used, and that the celebrity would deliver increasing payments throughout the process, with the first payment occurring after two weeks of painting. One week into the process, after the painting had begun, the celebrity died. Her family demanded that the artist continue with the painting, using photographs as a substitute for the daily sessions. Is the artist required to complete a painting of the celebrity? Answers: Yes, because he can complete it by relying on pictures of the celebrity. Yes, because he had begun painting the celebrity. No, because the celebrity died after only one week. No, because no payment had yet occurred.

Answer choice C is correct. If an offer has actually been accepted, death of the offeror does not automatically terminate the contract. The contract may still be enforceable unless there is some reason, such as impracticability, that warrants a discharge of the obligation. Here, the woman has died, so she cannot sit for the painting for the two months. Once the woman died, it became impracticable (and impossible even) to complete the painting under the terms of the contract. Therefore, the artist is not required to complete a painting of the celebrity. Answer choice A is incorrect because the contract did not specify the use of another method, and the facts state that the artist only uses the old-fashioned and rarely used style of painting. The family could not alter the terms of the contract after the celebrity's death. Answer choice B is incorrect because although the artist's performance reflects acceptance of the offer, the obligation is discharged upon the celebrity's death. Answer choice D is incorrect. Although payment had not yet occurred, that is not a valid reason to not enforce an otherwise enforceable contract. The first payment was not yet due under the contract.

Question 6355 Due to recent financial difficulties, a man asked his brother if he would be willing to loan him $3,000. The brother agreed to make the $3,000 loan to the man. Under the terms of their agreement entered into on December 31, the man would be responsible for making monthly payments of $125 plus interest for the next two years at the beginning of each month, starting January 1. Pursuant to the agreement with his brother, the man made payments each month for the first six months. However, he failed to make the agreed-upon payments for July and August. On August 30, the man told his brother that he could no longer afford to repay him. The brother filed suit against the man on August 31 for breach of contract. What amount, if any, will the brother be able to recover on August 31? Answers: $2,250 plus interest, the amount outstanding on the loan because the man repudiated the contract. $375 plus interest, the amount owed for July, August, and September. $250 plus interest, the amount owed for July and August. Nothing, because the agreement did not contain an acceleration clause.

Answer choice C is correct. If at the time of a breach the only remaining duties of performance are (i) those of the party in breach and (ii) for the payment of money in installments not related to one another, then breach by nonperformance as to less than the whole, whether or not accompanied or followed by a repudiation, does not give rise to a claim for damages for total breach and is a partial breach of contract only. In this case, on August 31, the brother is only entitled to recover the amount due for July and August, as the other installments are not due yet. Answer choice A is incorrect. Even though the man repudiated the entire contract by telling his brother he could no longer afford to repay him, the brother cannot recover the entire amount owed under the agreement and is limited to the installments that are currently outstanding as of August 31. Answer choice B is incorrect because the payment for September is not due yet. Answer choice D is incorrect. Although an acceleration clause would be required for the brother to recover the entire amount of the loan, the absence of an acceleration clause does not preclude the brother from recovering the amount currently due and outstanding under the agreement.

Question 6399 A grandson asked to borrow $2,000 from his grandmother. She told him that she did not have the money but that she would ask a friend to lend him the money. The friend agreed on the condition that the grandmother would guarantee the loan. After the grandmother sent an unsigned letter to her friend that outlined the terms of the loan and her agreement to guarantee its repayment, the friend made the loan. When the loan came due and the grandson was unable repay it, the friend demanded that the grandmother make good on her promise. When she refused to do so, the friend filed an action in small claims court against the grandmother. What is the grandmother's best defense to this action? Answers: The friend did not give the grandmother notice that he had accepted her offer to guarantee the loan. The Statute of Frauds prevents enforcement of the grandmother's promise because of the amount of the loan. The grandmother did not sign the letter containing her promise to guarantee the loan. The grandmother did not receive consideration in exchange for her promise to guarantee the loan.

Answer choice C is correct. Suretyship is a three-party contract, wherein one party (the surety) promises a second party (the obligee) that the surety will be responsible for any debt of a third party (the principal) resulting from the principal's failure to pay as agreed. Under the Statute of Frauds, a suretyship contract, or contract to answer for the debt of another, must be signed by the party to be charged. Here, because the grandmother had not signed the letter that contained her promise to act as a surety of the loan, the Statute of Frauds would prevent enforcement of her promise. Answer choice A is incorrect. Although generally an acceptance must be communicated to the offeror, because the grandmother's offer did not restrict the manner of acceptance, the friend's acceptance by performance (i.e., lending the money to the grandson) was permissible. With respect to an offer of suretyship, the surety does not need to be notified of the obligee's performance unless the terms of the offer specifically require it. Answer B is incorrect. Although a contract for the sale of goods must be in writing if the purchase price is $500 or more, there is no monetary limitation on an oral contract that does not fall within the confines of the UCC, such as the loan agreement in this case. Answer choice D is incorrect because the grandmother did not have to receive a benefit in order for there to be consideration under the contract. Consideration requires that some legal detriment be bargained for in order for a contract to exist. The exchange must be of some benefit to the promisor or detriment to the promisee. Here, the payment of $2,000 to the grandson was a detriment to the friend as promisee.

Question 6130 Upon the completion of an interview, an attorney offered an assistant $3,000 each month to perform administrative tasks at his office; he emailed her an offer confirming these terms. She responded via email, stating that she accepted the offer but inquired as to whether she would have to work in the office or could work remotely, expressing that the option to work from home was essential to her. The parties had discussed in the interview that the assistant would likely work in the office but that the attorney would be open to her working remotely at some point. The emailed offer did not address this issue. The attorney did not respond that day but went out and bought the assistant a laptop conforming to her preferences and many office supplies that she had requested. The next day, the attorney emailed her to confirm that he might allow her to work remotely after she demonstrated success within an office environment. She emailed him back that she no longer wished to work with him because she preferred to work from home right away. The attorney threatened to sue her for breach of contract. Under common-law principles, have the attorney and assistant entered into an enforceable contract? Answers: Yes, because the assistant emailed him back to accept the offer. Yes, because the attorney's offer did not propose that the assistant could work from home. No, because the assistant suggested additional terms in her response. No, because the assistant had a misunderstanding as to the terms of the contract.

Answer choice C is correct. The acceptance must mirror the terms of the offer. Any change to the terms of the offer, or the addition of another term not found in the offer, acts as a rejection of the original offer and as a new counteroffer. Mere suggestions or inquiries, including requests for clarification or statements of intent, made in a response by the offeree do not constitute a counteroffer. However, the addition of a term that is essential to the party likely indicates that the response was a counteroffer, not a mere inquiry. In this case, the assistant's acceptance of the attorney's offer was conditional upon her being able to work from home. Accordingly, it was not really an acceptance, it was a rejection and counteroffer. Accordingly, answer choice A is incorrect. Answer choice B is incorrect because it represents a fact that supports that the assistant's question was merely a means for clarification. However, it is not in and of itself a reason for enforcing the contract. Answer choice D is incorrect because she did not misunderstand the terms of the contract. Rather, she proposed an additional term.

Question 6144 A company leased office space in a downtown building and subsequently entered into a written contract with a supplier to purchase furniture for the office. Among the provisions in the contract was the following: "This document contains the entire and final agreement of the parties. It supersedes any prior agreements, understandings, or negotiations, whether written or oral." A dispute subsequently arose over the tables and desks delivered by the supplier. The contract called for "cherry tables and desks" of designated designs. The company contended that the word "cherry" indicated the type of wood that the tables and desks were made of. The supplier, having delivered tables and desks made of a less expensive wood and finished with a cherry veneer, asserted that the use of the word "cherry" referred to the appearance of these items and did not require that the furniture be made solely of cherry wood. In the litigation of this dispute, the company sought to introduce a statement made by the supplier during negotiations that the tables and desks were of "solid-wood construction." In determining whether the contract was a total integration of the agreement between the company and the supplier, which of the following rules should the court apply? Answers: The four-corners rule The "naturally omitted" rule The "certainly included" rule The plain-meaning rule

Answer choice C is correct. The contract is for the sale of particular pieces of furniture, which are goods, and therefore it is governed by the Uniform Commercial Code (UCC). Under the UCC, a court, in determining whether a contract constitutes a total integration of the parties' agreement, should generally treat a written agreement as only a partial integration of the parties' agreement unless the court can conclude that the parties' "certainly would" have included the term in the written agreement. Answer choice A is incorrect. Under the common-law four-corners rule, a court is required to look only to the wording of the document itself (i.e., within its "four corners") in determining whether the parties intended the document to be a total integration of their agreement. However, this rule does not apply to contracts, such as this one, that are governed by the UCC. Answer choice B is incorrect. The "naturally omitted" rule of the Restatement (Second) of Contracts requires a court to consider whether an extrinsic term of an agreement would "naturally be omitted" from a writing in determining whether the parties intended the document to be a total integration of their agreement. However, this rule does not apply to contracts, such as this one, that are governed by the UCC. Answer choice D is incorrect because the plain-meaning rule, which provides that the objective definitions of contract terms control the meaning of the contract, may be applicable in ascertaining whether to admit extrinsic evidence despite the written agreement being a total integration. However, this rule is not applicable in determining whether, in the first instance, the written agreement is a total integration.

Question 7021 An organic produce supplier sent her produce catalog to a local restaurant on April 15. The catalog came with a signed letter stating: "I will supply you with as many of the items in the enclosed catalog as you order before August 1 of this year, and I assure you that this offer and the prices provided in the catalog will remain firm until August." The supplier received no reply. In June, the supplier's tomato crop was infested with a fungus that decimated her harvest. She was left with half the tomatoes she had expected to harvest. On June 15, the supplier sent the restaurant a signed letter stating that the price for the tomato crop was now twice the price listed in the catalog. On July 1, the restaurant sent the supplier an order for tomatoes. However, the restaurant demanded the tomatoes at the price listed in the catalog. Has a contract been formed for the sale of the tomatoes to the restaurant at the catalog price? Answers: No, because the destruction of the tomatoes supports a defense of impracticability. No, because the restaurant offered no consideration to make the original offer irrevocable. Yes, because the original offer was irrevocable on June 15 and on July 1. Yes, because the restaurant exercised its power of acceptance within a reasonable time.

Answer choice C is correct. Under the UCC, an offer to buy or sell goods is irrevocable if the offeror is a merchant, there is an assurance that the offer is to remain open, and the assurance is contained in a signed writing from the offeror. The irrevocability of a firm offer cannot exceed three months unless the offeree gives consideration to validate it beyond the three-month period. Here, the original offer meets these requirements, and both the date the supplier tried to change the price (June 15) and the date the restaurant accepted the offer (July 1) fall within the three-month period of irrevocability. Therefore, the offer was still open and irrevocable when the restaurant accepted it on July 1. Answer choice A is incorrect because the question asks whether a contract has been formed, and the answer is yes under the UCC firm offer rule. Impracticability would come into play only if a contract had been formed and the question asked whether the supplier had a defense to excuse nonperformance. Answer choice B is incorrect because both the date the supplier tried to change the price and the date the restaurant accepted the offer fall within the three-month period of irrevocability. Therefore, the offer was still open and irrevocable without consideration when the restaurant accepted it on July 1. Answer choice D is incorrect because an offer can be accepted only when it is still outstanding, regardless of whether the acceptance occurred in a reasonable time. If the offer here had been revocable, the fact that the restaurant tried to accept the offer after its revocation but within a reasonable time would not create an enforceable contract.

Question 7039 A refrigeration-unit manufacturer contracted with a kitchen appliance store to sell and deliver 100 refrigeration units to the store at a price substantially lower than market value. The written and signed contract included the term "F.O.B. kitchen appliance store, on or before March 30." Due to an unforeseen strike by the shipping company, the manufacturer delivered the units to the store on April 18. The store suffered no material harm due to the delay. The refrigeration appliance industry generally allows appliance manufacturers a 30-day leeway for any contractually specified time of delivery, unless such leeway is expressly forbidden by the contract. If the store brings suit against the manufacturer for breach of contract, which of the following facts provides the manufacturer with the strongest defense? Answers: The delay was caused by an unforeseeable strike. The manufacturer believed that due to the price at which it offered the refrigeration units, the store would accept a late delivery. There is evidence of a trade usage in the refrigeration appliance industry allowing a 30-day leeway for appliance deliveries. The store suffered no material harm from the delay.

Answer choice C is correct. Under the UCC, even if the terms of a written contract for the sale of goods appear to be unambiguous, a party may explain or supplement the terms by evidence of trade usage or course of dealings or performance. Trade usage is any practice or method of dealing in the particular business or industry that is practiced with such regularity so as to justify an expectation that it will be practiced in the instant case. Here, the manufacturer's strongest argument will be that a trade usage applies, and that the terms of the contract should be supplemented by the trade usage in the industry allowing a 30-day leeway for appliance deliveries. Answer choice A is incorrect. While it is true that unforeseeable strikes have been found to excuse performance, here, performance was not impracticable; it was simply delayed. The manufacturer itself ultimately delivered the units to the store, and there is no indication that they could not have done so by the agreed-upon delivery date. Therefore, although the foreseeability of the strike will be relevant to calculating damages, it is not the manufacturer's strongest defense against the breach of contract claim. Answer choice B is incorrect because the manufacturer will not be excused for breaching a contract simply because it believed offering low prices would encourage the store to accept even a nonconforming performance. Answer choice D is incorrect because material harm is not a necessary element of bringing a breach of contract action, though it may be relevant to damages.

Question 2790 A mining company contracted with a railroad to transport 10,000 tons of coal from the company's mines to a power company at a cost of $100,000. The railroad told the mining company that the coal would arrive at the power company on June 1, but the contract contained a clause that the railroad would not be liable for any losses suffered by the mining company as a result of a late shipment. The railroad was aware that the mining company had contracted with the power company to deliver the coal on June 1, and pursuant to standard industry custom, the price to be paid by the power company decreased by $1 per ton for each day that the coal was late. The shipment of coal did not reach the power company until June 11, and the railroad had no justification for the 10-day delay. Because of the delay, the mining company lost $100,000 in revenue from the sale. The mining company filed suit against the railroad for breach of contract, claiming $100,000 in damages. Is the mining company likely to succeed in its claim? Answers: Yes, because the damages that the mining company would suffer from the railroad's delay were known to the railroad prior to shipment of the coal. Yes, because consequential damages cannot be excluded by a merchant. No, because the claimed damages are disproportionate to the original contract price between the railroad and the mining company. No, because the contract between the mining company and the railroad protected the railroad from losses suffered by the mining company due to a late shipment.

Answer choice D is correct. Although a party may be liable for consequential damages of the other party to a contract where those damages are foreseeable, a party may eliminate that liability through an agreement with the other party. Here, while the railroad was aware of the mining company's liability for failing to supply the power company with coal on June 1st, the contract between the railroad and the mining company eliminated the railroad's liability for any losses suffered by the mining company due to late delivery. Answer choice A is incorrect because the parties to a contract are generally free to allocate responsibility for consequential damages as they see fit. So even though the mining company's liability due to a late shipment was known to the railroad, the railroad expressly disavowed responsibility for that liability. Answer choice B is incorrect because, although the UCC does prohibit the limitation or exclusion of liability for consequential damages, this prohibition applies only where limitation or exclusion is unconscionable, and a commercial loss is not prima facia unconscionable. In addition, the contract at issue (i.e., between the railroad and the mining company) is not a sale of goods and hence not governed by the UCC. Answer choice C is incorrect because, while the lack of proportionality between the contract price and the amount of consequential damages may be an underlying factor in determining whether such damages are foreseeable, proportionality alone is not determinative as to whether such damages are foreseeable. In any case, the parties may generally allocate responsibility for consequential damages as they see fit.

Question 5987 At the auction of a rare automobile that had been owned by an infamous criminal, two individuals, a museum owner and a private collector, became involved in a bidding war. After a protracted exchange of bids, the museum owner declined the auctioneer's invitation to bid a higher amount than the private collector had. However, as the auctioneer was bringing down his hammer to announce the sale of the automobile to the private collector, the museum owner called out a higher bid before the hammer struck the sounding block on the auctioneer's desk. The auctioneer acknowledged the museum owner's bid and called for other bids. A third party placed a bid, which unmatched by either the museum owner or the private collector, was subsequently accepted by the auctioneer as the winning bid. The museum owner and private collector challenged the third party's ownership of the automobile. Who is the rightful owner of the automobile? Answers: The private collector, based on the private collector's last bid before the fall of the hammer. The private collector, based on the museum owner's refusing the auctioneer's invitation to bid prior to the fall of the hammer. The museum owner, based on the museum owner's bid made while the hammer was falling. The third party.

Answer choice D is correct. An auction sale is complete when the auctioneer announces its end, such as by the fall of the auctioneer's hammer or in any other customary way. When a bid is made contemporaneously with the falling of the hammer, the auctioneer may, at his discretion, treat the bid as continuing the bidding process or declare the sale completed at the fall of the hammer. In this case, the museum owner made his bid contemporaneously with the falling of the hammer. The auctioneer chose, at his discretion, to treat the museum owner's bid as continuing the bidding process. The third party subsequently made a bid that was unmatched. Therefore, because the third party's bid was accepted by the auctioneer as the winning bid, the third party is entitled to the automobile. Answer choice A is incorrect. Although the auctioneer could have elected to award the automobile to the private collector based on the collector's last bid made before the fall of the auctioneer's hammer, the auctioneer was not required to do so. Answer choice B is incorrect. Although the museum owner did decline the auctioneer's invitation to bid prior to the fall of the hammer, this failure to bid at that time did not preclude the auctioneer from recognizing the museum owner's subsequent bid. Answer choice C is incorrect because, although the auctioneer had the option to recognize the museum owner's bid that was made while the hammer was falling, the auctioneer could not automatically accept this bid as the final bid. Instead, having recognized this bid, the auctioneer was required to treat it as a continuation of the bidding.

Question 6336 A produce wholesaler sent a written offer to a farmer to purchase all of the corn produced by the farmer for a period of two years. Excited at the prospect of having a guaranteed sale for all of his corn, the farmer immediately communicated his acceptance to the wholesaler. The wholesaler and farmer entered into a written contract reflecting the basic terms set forth in the wholesaler's offer. Six months after the contract was executed, the wholesaler determined that, while the farmer's corn was returning a profit, the farmer's corn was not selling as well as corn that the wholesaler could acquire from other sources. Nevertheless, the wholesaler contacted the farmer and informed him that he would no longer purchase the farmer's corn and would be buying his corn from another source. If the farmer sues the wholesaler for breach of contract, is he likely to prevail? Answers: No, because the contract did not contain a specific quantity term. No, because the wholesaler no longer needed the farmer's goods. Yes, because the farmer relied on the wholesaler's promise. Yes, because the wholesaler did not act in good faith.

Answer choice D is correct. An output contract is a contract under which a seller agrees to sell all that she manufactures of a product to the buyer. Here, the wholesaler still needed corn, and was choosing instead to purchase it from another party. There is no indication that this situation would allow the wholesaler to cancel the contract without violating the good-faith requirement of the contract. Therefore, the farmer will likely succeed in a breach of contract action against the wholesaler. Answer choice A is incorrect because output contracts satisfy UCC formation requirements even without naming specific quantities or a specific price. Answer choice B is incorrect. The wholesaler here still needed the farmer's goods; he just chose to buy them elsewhere. Therefore, he could not cancel the contract in good faith on these grounds. Answer choice C is incorrect. The farmer and wholesaler entered into a valid and enforceable output contract supported by consideration. Accordingly, the farmer does not need to show reliance on the wholesaler's promise (i.e., promissory estoppel) to prevail.

Question 6021 A private port authority contracted with a crane operating company to assist with loading and unloading containers from ships docked at the port. One of the company's cranes was defectively manufactured. Due to this defect, a container was dropped. The container tumbled down a hill, crashed through a fence, and struck a passerby. The passerby sued the port authority alleging negligence. Neither the passerby nor the port authority notified the crane operating company of this lawsuit. The port authority settled its claim with the passerby before trial for a reasonable amount. The port authority seeks to recover the cost of the settlement from the crane operating company under a breach of contract action. Of the following, which would be the crane operating company's best defense? Answers: The crane operating company was not notified of the lawsuit. Damages for personal injury cannot be recovered in a breach of contract action. The port authority settled the lawsuit rather than litigating the matter to a final judgment. The settlement was not reasonably foreseeable at the time the contract was formed.

Answer choice D is correct. Consequential damages are recoverable in a breach of contract actions only if they were reasonably foreseeable at the time that the contract was entered into. While it is arguable that, although the actual means by which the passerby was injured was not foreseeable, the fact that a person could be injured by a dropped container and therefore recover in a negligence action was foreseeable, the other answer choices present arguments that would not protect the crane operating company from contractual liability regardless of whether the passerby's injuries and the subsequent lawsuit and its settlement were foreseeable. Answer choice A is incorrect because, while the failure to provide the crane operating company with notice of the lawsuit would protect the crane operating company from liability directly imposed on a party to the lawsuit, this lack of notification would not protect the crane operating company from contractual liability to the port authority if the settlement was reasonably foreseeable at the time that the contract was formed. Answer choice B is incorrect because, while generally damages attributable to a personal injury are recoverable in a tort action rather than a breach of contract action, at issue in the action initiated by the port authority is the foreseeability of those damages and the subsequent settlement by the port authority of a claim from those damages. Answer choice C is incorrect because the fact that the port authority elected to settle the negligence action brought by the passerby for reasonable amount rather than litigate the matter to a final judgment does not prevent the port authority from seeking to recover the cost of the settlement as consequential damages.

uestion 7036 On November 1, the owner of a yacht posted a flier at a local coffee shop reading "Yacht for Sale: Make me an offer!" The flier also included the owner's phone number. A buyer called the owner on November 3 to ask how much the owner wanted for the yacht. The owner said, "Well, I'd hate to part with it for less than $55,000, but if you can pay me $50,000 by November 20, I suppose I'd sell it to you. I'll hold onto the yacht for you until then." Elated, the buyer took steps to obtain a loan by November 20. On November 15, a second buyer called the owner and offered to buy the yacht for $60,000. The owner immediately accepted, and the second buyer picked up the yacht the next day. On November 20, having obtained a loan, the first buyer visited the owner with a check for $50,000. The first buyer then learned the owner had already sold the yacht. Can the first buyer bring a successful suit against the owner for breach of contract? Answers: No, because the owner's statement to the first buyer was only an invitation to deal. No, because the second buyer offered more money for the yacht than the first buyer agreed to pay. Yes, because the owner promised to keep the offer open for a specific period of time. Yes, because the owner's offer to the first buyer was still outstanding on November 20.

Answer choice D is correct. In general, an offer can be revoked by the offeror at any time prior to acceptance, and an offer is revoked when the offeror makes a manifestation of an intention not to enter into the proposed contract. Offers are open for a reasonable time, and an offer can be accepted at any time before the offer is terminated. Here, because the offeror said that he would sell the yacht to the buyer if he could pay by November 20, acceptance by November 20 would be acceptance within a reasonable amount of time. The buyer could have revoked the offer before November 20 (the contract did not qualify as an option, as explained below), but the owner never actually revoked the offer to the first buyer. Therefore, the first buyer could still accept the offer on November 20, and the owner can be liable for breaching the contract. Note that this would not qualify as a constructive revocation because the buyer did not learn of the prior sale before he accepted the offer. Answer choice A is incorrect because the owner's statement to the first buyer was sufficiently specific (as to the terms, his intent to make an offer, and who may accept it) to constitute an offer. Answer choice B is incorrect because the fact that the second buyer offered more money is irrelevant to whether a contract was formed between the first buyer and the owner. Answer choice C is incorrect. If the option is a promise not to revoke an offer to enter a new contract, the offeree must generally give separate consideration for the option to be enforceable. Here, the first buyer did not offer consideration to keep the offer open. Even if the owner were considered a merchant in this transaction, the offer to keep the option open was not in a signed writing, and thus the firm offer rule would not apply. Therefore, the offer remained revocable. The reason that the buyer will succeed in this action is not because the offer was irrevocable as an option, but because the owner failed to effectively revoke the offer.

Question 6334 A new men's apparel company entered into a contract with a well-known former mayor to make appearances at different retail stores throughout the country to promote the company's line of men's suits. Knowing that the former mayor's sponsorship had helped other new companies in the area build a consumer base, the company agreed to pay the former mayor $10,000 per appearance. Prior to the former mayor's first appearance, a news story was released that alleged the company engaged in discriminatory practices in the workplace. Despite the company's truthful assertions that the news story was completely false, the former mayor refused to make any appearances under the contract. The company was unable to find another famous public figure to make the appearances and, as a result, its line of men's suits was a complete failure. The company subsequently sued the former mayor for breach of contract. At trial, the company's attorney presented reliable uncontested evidence calculating the lost profits sustained by the company due to the former mayor's failure to make the appearances. Will the company likely be able to recover damages for lost profits? Answers: No, because the damages for lost profits were not foreseeable. No, because the lost profits of a new business cannot be recovered. Yes, because the former mayor anticipatorily repudiated the contract. Yes, because the lost profits of a new business are recoverable if proved with reasonable certainty.

Answer choice D is correct. To recover damages, a plaintiff must prove the dollar amount of the damages with reasonable certainty. Courts are hesitant to award damages for lost profits, as they are difficult to prove. When lost profits are considered too speculative, such as with a new venture, courts often limit a party's recovery to reliance damages (i.e., reasonable expenditures made in connection with the contract). However, in this case, the facts provide that the company's attorney presented reliable and uncontested evidence regarding the company's lost profits due to the former mayor's breach. This evidence is sufficient to prove those damages with reasonable certainty, especially as the evidence was uncontested. Accordingly, although the company is a new business, on these facts, it can recover its lost profits. Answer choice A is incorrect. Damages are recoverable if they were the natural and probable consequences of breach. Here, it was foreseeable that the former mayor's failure to appear at the retail stores would result in significantly lower sales numbers for the company's new line of suits. Answer choice B is incorrect because lost profits of a new business can be recovered if proved with reasonably certainty. Answer choice C is incorrect. Although the former mayor's failure to appear constituted an anticipatory repudiation of the contract, that fact would entitle the company to damages, but not necessarily lost profits. The reason lost profits are recoverable here is because they were proved with reasonable certainty by the company's attorney. Therefore, answer choice D is the best answer choice.

Question 4224 A plastics manufacturer saw an advertisement for a plastic extruding machine. The manufacturer contacted the seller and made arrangements to inspect the machine at the seller's place of business. The manufacturer walked around the machine once and stated: "Yes, this looks like what I need." When the manufacturer asked the price, the seller stated a price that was less than half the amount a similar, functioning, used machine commanded on the market. The manufacturer was surprised at the low price, but did not inquire as to the reason. The seller encouraged the manufacturer to perform a closer inspection before finalizing the purchase and offered to open the motor housing so that the motor could be examined, but the manufacturer declined. The parties completed the sale. The manufacturer transported the extruding machine to his factory. When it arrived, he first learned that the motor was burned out and required complete replacement, as was readily apparent upon visual inspection of it. Replacement of the motor would cost roughly the amount the manufacturer had paid for the machine. The manufacturer contacted the seller to return the machine, but the seller refused. The manufacturer filed suit against the seller. Will the manufacturer prevail? Answers: Yes, because the seller violated the implied warranty of merchantability by selling a machine with a burned-out motor. Yes, because the manufacturer's unilateral mistake regarding the condition of the machine was caused by the seller. No, because the seller made no claims regarding the operability of the machine. No, because the manufacturer waived any implied warranties by failing to inspect the machine.

Answer choice D is correct. Under UCC Article 2, a warranty of merchantability is implied whenever the seller of goods is a merchant. To be merchantable, goods must be fit for their ordinary purpose and pass without objection in the trade. A breach of this warranty must have been present at the time of the sale. However, if the buyer, before entering into the contract, has examined the goods as fully as the buyer desires, or has refused to examine the goods, there is no implied warranty with respect to defects that an examination would have revealed to the buyer. Here, the manufacturer declined to closely inspect the machine, even after learning of the unusually low sales price. Had he done so, he would have discovered the damaged motor when he opened the motor housing, which the seller had offered to do. Because the manufacturer refused to examine the goods, he waived any implied warranty of merchantability that would have otherwise attached to the sale. Answer choice A is incorrect because the manufacturer waived the implied warranty of merchantability when he declined to inspect the machine. Answer choice B is incorrect because the seller did not cause the manufacturer's unilateral mistake; rather, the seller offered to open the motor housing, and the manufacturer declined the offer. Answer choice C is incorrect because the warranty of merchantability is implied in all sales of goods and need not be expressly stated by the seller. If the defect could not have been uncovered by a reasonable inspection, the manufacturer would have had a valid claim against the seller for violation of the implied warranty of merchantability, even though the seller had not expressly made any such claims.

Question 6381 A man was moving to another state and decided that he wanted to give away some of his belongings. He knew his brother had always expressed interest in an antique desk. The man called his brother and said, "I'm going to be moving in two weeks. I would like to give you the antique desk as a gift. I'll drop it off at your house on my way out of town." The brother told the man that he was very grateful for the gift and was looking forward to having the desk in his home office. The brother immediately disposed of his old desk and made room for the antique one. A couple of days later, an appraiser, who was a friend of the man, visited the man's house for dinner. While at his house, he saw the antique desk and informed the man that it was worth well over $20,000. The man decided to keep the desk and did not drop it off at the brother's house on his way out of town. The brother brought suit against the man to recover the antique desk. If the court finds in favor of the man on these facts, what is the most likely reason? Answers: A promise to make a gift in the future cannot be enforced. The brother did not rely to his detriment on the man's promise. The man's promise was not in writing. The man's refusal to give the piece of furniture did not cause injustice.

Answer choice D is correct. Under the doctrine of promissory estoppel, a party's promise to make a gift is enforceable if the donor should reasonably expect the promise to induce detrimental reliance by the donee, the promise actually induces such reliance, and the failure to enforce the promise will cause injustice. In this case, the man's promise to give the piece of furniture to his brother will only be enforced if enforcement is necessary to avoid injustice. Because the facts indicate that the brother did rely on the promise by disposing of his desk, and that the court finds in favor of the man anyway, the court must have concluded that the man's failure to give the desk as promised did not cause injustice. Answer choice A is incorrect because a promise to make a gift in the future can be enforced if the requirements for promissory estoppel are met. Answer choice B is incorrect because the brother arguably did rely to his detriment on the man's promise by immediately disposing of his old desk. Therefore, on these facts, this would not be a likely reason to find in favor of the man. Accordingly, answer choice D is a better answer. Answer choice C is incorrect because a writing would not be required to enforce a promise to make a gift under the doctrine of promissory estoppel. Therefore, the lack of a writing would not be dispositive here, and answer choice D is the better choice.

Question 6349 On January 8, a liquor company sent a signed, written offer to a retailer containing the following: "We will sell you our last 500 cases of our caffeine-infused vodka for $100 per case." On that same day but unknown to either party, the state enacted a law prohibiting the sale or distribution of the caffeine-infused vodka due to a number of serious health risks connected to the product. Upon receiving the liquor company's offer, the retailer decided to take a couple of days to contemplate the offer. On January 15, after not hearing from the retailer, the liquor company decided it was no longer willing to sell the vodka to the retailer, but it did not contact the retailer. On January 17, the retailer decided that it was not interested in the vodka and placed a letter rejecting the offer in the mail. However, after hearing about the popularity of the vodka from a number of patrons that same day, the retailer immediately mailed a signed acceptance to the liquor company. The liquor company received the retailer's acceptance on January 20. Due to an error at the post office, the retailer's initial rejection was not received until January 21. The retailer and the liquor company both learned of the state law on January 22. Which of the following is an accurate statement regarding the relationship between the retailer and the liquor company as of January 23? Answers: No contract was formed because the liquor company's offer was revoked on January 15. No contract was formed because the retailer's rejection was effective on January 17. An enforceable contract was formed on January 20. A contract was formed on January 20, but it is not enforceable due to illegality.

Answer choice D is correct. Under the mailbox rule, an acceptance that is mailed within the allotted response time is effective when sent (not upon receipt), unless the offer provides otherwise. Because acceptance becomes valid when sent, a properly addressed letter sent by the offeree operates as an acceptance when mailed, even though the offeror has not yet received the notice. If a communication is sent rejecting the offer, and a later communication is sent accepting the contract, then the mailbox rule will not apply, and the first one to be received by the offeror will prevail. Here, the retailer placed a letter in the mail rejecting the offer on January 17. However, later that same day, it mailed an acceptance of the offer to the liquor company. Accordingly, the mailbox rule does not apply. Because the liquor company received the retailer's acceptance first, a contract was formed on January 20. However, the state law made the sale or distribution of the particular vodka illegal. As a result, the contract between the liquor company and the retailer is not enforceable due to illegality. Answer choice A is incorrect. A revocation may be made in any reasonable manner and by any reasonable means, and it is not effective until communicated. In this case, although the liquor company determined that it would not sell the vodka to the retailer, it did not communicate that decision to the retailer. Accordingly, the liquor company's offer was not revoked on January 15. Answer choice B is incorrect. A rejection is usually effective upon receipt, not mailing. Thus, the retailer's rejection was not effective on January 17 when it was mailed. Answer choice C is incorrect. Although a contract was formed on January 20, the contract is not enforceable due to illegality. Therefore, this is an incorrect statement of the relationship between these parties.


संबंधित स्टडी सेट्स

Personal Finance Midterm (chapter 1)

View Set

International Business Final Exam

View Set

Decreased Circulating Oxygen/ Anemia

View Set

Chapter 26: Management of Patients with Dysrhythmias and Conduction Problems compact

View Set

Team Deutsch 1 Vocabulaire Lektion 1 Hallo, wie geht's?

View Set